You are on page 1of 58

Total Marks : 200

Online Prelims TEST - 18 (SUBJECT)


( InsightsIAS Mock Test Series for UPSC Preliminary Exam 2020 ) Mark Scored : 61.33

1 Consider the following statements


1. Pitt’s India Act of 1784 empowered the board of control to supervise and direct all operations of the
civil and military government.
2. Charter Act of 1813 ended the activities of the East India Company as a commercial body, which
became a purely administrative body.

Which of the statements given above is/are correct?


A. 1 only
B. 2 only
C. Both 1 and 2
D. Neither 1 nor 2

Your Answer : C
Correct Answer : A

Answer Justification :
1 3
27
Pitt’s India Act of 1784 empowered the Board of Control to supervise
8 25and direct all
63British possessions in India.
operations of the civil and military government or revenues of the
8
Hence Statement 1 is correct. -
.c om
Charter Act of 1833 ended the activities of the East
a il India Company as a commercial body,
which became a purely administrative body. It providedm that the company’s territories in India were
@gsuccessors’. Hence Statement 2 is incorrect.
held by it ‘in trust for His Majesty, His heirs5and
2 60
1
2 Consider the following statements r 75
regarding Government of India Act, 1919
m
- bicameralism and direct elections in the country.
1. It introduced, for the first time,
o y
yR
2. It provided for the establishment of a public service commission.
3. It separated, forjothe first time, provincial budgets from the Central budget
i t un
Which Mofr the statements given above is/are correct?
A. 1 only
B. 2 only
C. 2 and 3 only
D. 1,2, and 3

Your Answer : D
Correct Answer : D

Answer Justification :

All the statements given above are correct.

Government of India Act of 1919

On August 20, 1917, the British Government declared, for the first time, that its objective was the
gradual introduction of responsible government in India.

prelims.insightsonindia.com 1
© Insights Active Learning | All rights reserved - 156493. You may not reproduce, distribute or exploit the contents in any form without
written permission by copyright owner. Copyright infringers may face civil and criminal liability
Total Marks : 200
Online Prelims TEST - 18 (SUBJECT)
( InsightsIAS Mock Test Series for UPSC Preliminary Exam 2020 ) Mark Scored : 61.33

The Government of India Act of 1919 was thus enacted, which came into force in 1921. This Act is
also known as Montagu-Chelmsford Reforms (Montagu was the Secretary of State for India and
Lord Chelmsford was the Viceroy of India).

It introduced, for the first time, bicameralism and direct elections in the country. Thus, the
Indian Legislative Council was replaced by a bicameral legislature consisting of an Upper House
(Council of State) and a Lower House (Legislative Assembly).

It provided for the establishment of a public service commission. Hence, a Central Public
Service Commission was set up in 1926 for recruiting civil servants.

It separated, for the first time, provincial budgets from the Central budget and authorised
the provincial legislatures to enact their budgets.

3 Consider the following statements


1. The Constituent Assembly held its first meeting on December 9, 1947.
1 3
2. The representatives of the princely states readily joined constituent assembly
5 27
2
Which of the statements given above is/are correct? 6 38
A. 1 only -8
B. 2 only
.c om
C. Both 1 and 2 a il
D. Neither 1 nor 2 gm @
5
Your Answer : A 2 60
1
Correct Answer : D 75
- mr
Answer Justification o :y
y R
n j o
Both the statements are incorrect.
i t u
Mr
The Constituent Assembly held its first meeting on December 9, 1946. The Muslim League
boycotted the meeting and insisted on a separate state of Pakistan.

The representatives of the princely states, who had stayed away from the Constituent
Assembly, gradually joined it. On April 28, 1947, representatives of the six states were part of
the Assembly.

After the acceptance of the Mountbatten Plan of June 3, 1947 for a partition of the country, the
representatives of most of the other princely states took their seats in the Assembly.

4 Consider the following pairs regarding Committees of the Constituent


Assembly
Committees Headed By
Minorities Sub-Committee J.B. Kripalani
Union Constitution Committee Sardar Patel
Committee on the Functions of the Constituent
Rajendra Prasad
Assembly

prelims.insightsonindia.com 2
© Insights Active Learning | All rights reserved - 156493. You may not reproduce, distribute or exploit the contents in any form without
written permission by copyright owner. Copyright infringers may face civil and criminal liability
Total Marks : 200
Online Prelims TEST - 18 (SUBJECT)
( InsightsIAS Mock Test Series for UPSC Preliminary Exam 2020 ) Mark Scored : 61.33

Which of the pairs given above is/are correct?


A. 2 only
B. 2 and 3 only
C. 1 only
D. None

Your Answer :
Correct Answer : D

Answer Justification :

The Constituent Assembly appointed a number of committees to deal with different tasks of
constitution-making.

Committee on the Functions of the Constituent Assembly – G.V. Mavalankar

Minorities Sub-Committee – H.C. Mukherjee


1 3
5 27
Union Constitution Committee – Jawaharlal Nehru 2
6 38
-8
5 Consider the following statements
.c om
il passed during the British Era were
1. With the commencement of the Constitution, all the acts
a
repealed.
@ gm
5 on August 1947.
2. All the parts of the constitution were enforced
0
6
12
5 is/are correct?
Which of the statements given above
r 7
-m
A. 1 only
B. 2 only
o y
C. Both 1 and 2 R
y
D. Neithern1jonor 2
rit u
M
Your Answer : D
Correct Answer : D

Answer Justification :

Both the statements are incorrect.

With the commencement of the Constitution, the Indian Independence Act of 1947 and the
Government of India Act of 1935, with all enactments amending or supplementing the latter Act,
were repealed.
The Abolition of Privy Council Jurisdiction Act (1949) was however continued.

Some provisions of the Constitution pertaining to citizenship, elections, provisional parliament,


temporary and transitional provisions, and short title contained in Articles 5, 6, 7, 8, 9, 60, 324, 366,
367, 379, 380, 388, 391, 392 and 393 came into force on November 26, 1949 itself. The remaining
provisions (the major part) of the Constitution came into force on January 26, 1950.

prelims.insightsonindia.com 3
© Insights Active Learning | All rights reserved - 156493. You may not reproduce, distribute or exploit the contents in any form without
written permission by copyright owner. Copyright infringers may face civil and criminal liability
Total Marks : 200
Online Prelims TEST - 18 (SUBJECT)
( InsightsIAS Mock Test Series for UPSC Preliminary Exam 2020 ) Mark Scored : 61.33

6 Consider the following statements


1. Constituent power of Parliament under Article 368 does not enable it to alter the basic structure of
the Constitution.
2. The structural part of the Constitution is, to a large extent, derived from the Government of India
Act of 1935
3. The term ‘Federation’ has been mentioned in the Constitution

Which of the statements given above is/are correct?


A. 1 and 2 only
B. 3 only
C. 1, 2 and 3
D. 2 and 3 only

Your Answer : A
Correct Answer : A

1 3
Answer Justification :
27
25
In the Kesavananda Bharati case1 (1973), the Supreme Court ruled that
6 38the constituent power of
-8
Parliament under Article 368 does not enable it to alter the ‘basic structure’ of the Constitution.
Hence Statement 1 is correct. o m
a il.c
mextent, derived from the Government of India
The structural part of the Constitution is, to a large
g
5 @
Act of 1935. The philosophical part of the Constitution (the Fundamental Rights and the Directive
0
respectively. Hence Statement 2 5
26
Principles of State Policy) derive their inspiration
is1correct.
from the American and Russian Constitutions

m r7
The term ‘Federation’ has
y - nowhere been used in the Constitution. Article 1, on the other
hand, describes India o
Ras a ‘Union of States’ which implies two things: one, Indian Federation is not
o y
the result of anjagreement by the states; and two, no state has the right to secede from the
federation.tu n
ri Hence Statement 3 is incorrect.
M
7 Consider the following statements
1. Parliamentary system is based on the doctrine of separation of powers between the two organs.
2. Indian Parliament is a sovereign body.

Which of the statements given above is/are correct?


A. 1 only
B. 2 only
C. Both 1 and 2
D. Neither 1 nor 2

Your Answer : A
Correct Answer : D

Answer Justification :

prelims.insightsonindia.com 4
© Insights Active Learning | All rights reserved - 156493. You may not reproduce, distribute or exploit the contents in any form without
written permission by copyright owner. Copyright infringers may face civil and criminal liability
Total Marks : 200
Online Prelims TEST - 18 (SUBJECT)
( InsightsIAS Mock Test Series for UPSC Preliminary Exam 2020 ) Mark Scored : 61.33

Both the statements are incorrect.

The parliamentary system is based on the principle of cooperation and co-ordination


between the legislative and executive organs while the presidential system is based on the
doctrine of separation of powers between the two organs.

Even though the Indian Parliamentary System is largely based on the British pattern,
there are some fundamental differences between the two. For example, the Indian Parliament
is not a sovereign body like the British Parliament. Further, the Indian State has an elected head
(republic) while the British State has hereditary head (monarchy).

8 Consider the following statements


1. Constitution can be amended by a simple majority of the Parliament in the manner of ordinary
legislative process.
2. Supreme Court is the guarantor of the fundamental rights of the citizens and the guardian of the
Constitution. 3 1
7
Which of the statements given above is/are correct? 252
8
A. 1 only 63
B. 2 only -8
o m
il.c
C. Both 1 and 2

ma
D. Neither 1 nor 2

@ g
Your Answer : B 0 5
Correct Answer : C 126
5
m r7
Answer Justification : -
oy
yR
Both the statements are correct.
u njo
rit
Some provisions of the Constitution can be amended by a simple majority of the Parliament
in theMmanner of ordinary legislative process. Notably, these amendments do not come under Article
368.

The Supreme Court is a federal court, the highest court of appeal, the guarantor of the
fundamental rights of the citizens and the guardian of the Constitution.

9 Which of the following is/are the features of Parliamentary Form of Government?


1. Dissolution of the lower House
2. Membership of the ministers in the legislature
3. Leadership of the prime minister or the chief minister

Select the correct answer using the code given below


A. 1 and 2 only
B. 2 only
C. 1 and 3 only
D. 1, 2 and 3

prelims.insightsonindia.com 5
© Insights Active Learning | All rights reserved - 156493. You may not reproduce, distribute or exploit the contents in any form without
written permission by copyright owner. Copyright infringers may face civil and criminal liability
Total Marks : 200
Online Prelims TEST - 18 (SUBJECT)
( InsightsIAS Mock Test Series for UPSC Preliminary Exam 2020 ) Mark Scored : 61.33

Your Answer :
Correct Answer : D

Answer Justification :

All the above are the features of the Parliamentary form of government.

The Constitution establishes the parliamentary system not only at the Centre but also in the states.
The features of parliamentary government in India are:

(a) Presence of nominal and real executives;

(b) Majority party rule,

(c) Collective responsibility of the executive to the legislature,

(d) Membership of the ministers in the legislature,


1 3
(e) Leadership of the prime minister or the chief minister, 5 27
2
6 38
-8
(f) Dissolution of the lower House (Lok Sabha or Assembly).

.c om
a il of the Indian State
10 Which of the following features reveal the secular character
gmon the ground of religion
1. The State shall not discriminate against any citizen
@
2. Every religious denomination or any of its0 5 shall have the right to manage its religious affairs
section
6
5 12
3. All minorities shall have the right to establish and administer educational institutions of their choice

m r7
- the code given below
Select the correct answer using
y
Ro
A. 1 and 2 only
B. 3 only
jo y
C. 1 and 3n only
t u
D. 1,r2i and 3
M
Your Answer : D
Correct Answer : D

Answer Justification :

All the above statements are correct.

The Constitution of India stands for a secular state. Hence, it does not uphold any particular
religion as the official religion of the Indian State. The following provisions of the Constitution
reveal the secular character of the Indian State:

(a) The term ‘secular’ was added to the Preamble of the Indian Constitution by the 42nd
Constitutional Amendment Act of 1976.

(b) The Preamble secures to all citizens of India liberty of belief, faith and worship.

prelims.insightsonindia.com 6
© Insights Active Learning | All rights reserved - 156493. You may not reproduce, distribute or exploit the contents in any form without
written permission by copyright owner. Copyright infringers may face civil and criminal liability
Total Marks : 200
Online Prelims TEST - 18 (SUBJECT)
( InsightsIAS Mock Test Series for UPSC Preliminary Exam 2020 ) Mark Scored : 61.33

(c) The State shall not deny to any person equality before the law or equal protection of the laws

(Article 14).

(d) The State shall not discriminate against any citizen on the ground of religion (Article 15).

(e) Equality of opportunity for all citizens in matters of public employment (Article 16).

(f) All persons are equally entitled to freedom of conscience and the right to freely profess, practice
and propagate any religion (Article 25).

(g) Every religious denomination or any of its section shall have the right to manage its religious
affairs (Article 26).

(h) No person shall be compelled to pay any taxes for the promotion of a particular religion (Article
27).

13 by the State
(i) No religious instruction shall be provided in any educational institution maintained
7
(Article 28). 52
3 82
86 language, script or culture
(j) Any section of the citizens shall have the right to conserve its distinct
-
om
(Article 29).

il .c
(k) All minorities shall have the right to establish andaadminister educational institutions of their
choice (Article 30). gm
0 5@
(l) The State shall endeavor to secure for
1 26all the citizens a Uniform Civil Code (Article 44).
r 75
y -m
11 Consider the following statements
1. In India, all citizens R
o
irrespective of the state in which they are born or reside enjoy the same
y
njorights of citizenship all over the country.
political and civil
u
2. The voting
ritage was reduced to 18 years from 21 years in 1989 by the 69th Constitutional
M Act of 1988.
Amendment
3. The Indian Constitution adopts universal adult franchise as a basis of elections to the Lok Sabha
and the state legislative assemblies.

Which of the statements given above is/are correct?


A. 1 and 2 only
B. 3 only
C. 1 and 3 only
D. 1, 2 and 3

Your Answer : D
Correct Answer : C

Answer Justification :

Universal Adult Franchise

prelims.insightsonindia.com 7
© Insights Active Learning | All rights reserved - 156493. You may not reproduce, distribute or exploit the contents in any form without
written permission by copyright owner. Copyright infringers may face civil and criminal liability
Total Marks : 200
Online Prelims TEST - 18 (SUBJECT)
( InsightsIAS Mock Test Series for UPSC Preliminary Exam 2020 ) Mark Scored : 61.33

The Indian Constitution adopts universal adult franchise as a basis of elections to the Lok
Sabha and the state legislative assemblies. Every citizen who is not less than 18 years of age
has a right to vote without any discrimination of caste, race, religion, sex, literacy, wealth, and so
on.

The voting age was reduced to 18 years from 21 years in 1989 by the 61st Constitutional
Amendment Act of 1988. The introduction of universal adult franchise by the Constitution-makers
was a bold experiment and highly remarkable in view of the vast size of the country, its huge
population, high poverty, social inequality and overwhelming illiteracy.14

Universal adult franchise makes democracy broad-based, enhances the self-respect and prestige of
the common people, upholds the principle of equality, enables minorities to protect their interests
and opens up new hopes and vistas for weaker sections.

12 Consider the following statements


1. Article 3 provides for formation of new states and alteration of areas, boundaries or
1 3names of
existing states.
5 27
3 82
2. Article 29 provides for right of minorities to establish and administer educational institutions
3. Article 48 provides for the separation of judiciary from executive 6
-8
Which of the statements given above is/are not correct? com
A. 1 and 2 only a il.
B. 2 and 3 only
@ gm
C. 3 only
6 05
D. 1 and 3 only 2 1
5
m r7
Your Answer : -
Correct Answer : B oy
j oyR
i t un
Answer Justification :

Mr
Article 3 provides for formation of new states and alteration of areas, boundaries or names of
existing states. Hence Statement 1 is correct.

Article 30 provides for right of minorities to establish and administer educational institutions.
Hence Statement 2 is incorrect.

Article 50 provides for the separation of judiciary from executive. Hence Statement 3 is
incorrect.

prelims.insightsonindia.com 8
© Insights Active Learning | All rights reserved - 156493. You may not reproduce, distribute or exploit the contents in any form without
written permission by copyright owner. Copyright infringers may face civil and criminal liability
Total Marks : 200
Online Prelims TEST - 18 (SUBJECT)
( InsightsIAS Mock Test Series for UPSC Preliminary Exam 2020 ) Mark Scored : 61.33

1 3
5 27
2
6 38
-8
o m
13 Consider the following statements il.c
g ma of individuals, and at the same time,
1. Liberty means the absence of restraints on the activities
providing opportunities for the development5of@individual personalities.
2. Equality means the absence of special 2 6 0
privileges to any section of the society, and the provision of
5 1
7
adequate opportunities for all individuals without any discrimination.

- mr
Which of the statementsygiven above is/are correct?
A. 1 only y Ro
B. 2 only njo
r
C. Bothit1uand 2
D.MNeither 1 nor 2

Your Answer : C
Correct Answer : C

Answer Justification :

Both the statements are correct.

The term ‘liberty’ means the absence of restraints on the activities of individuals, and at
the same time, providing opportunities for the development of individual personalities.

The Preamble secures to all citizens of India liberty of thought, expression, belief, faith and worship,
through their Fundamental Rights, enforceable in court of law, in case of violation.

The term ‘equality’ means the absence of special privileges to any section of the society,

prelims.insightsonindia.com 9
© Insights Active Learning | All rights reserved - 156493. You may not reproduce, distribute or exploit the contents in any form without
written permission by copyright owner. Copyright infringers may face civil and criminal liability
Total Marks : 200
Online Prelims TEST - 18 (SUBJECT)
( InsightsIAS Mock Test Series for UPSC Preliminary Exam 2020 ) Mark Scored : 61.33

and the provision of adequate opportunities for all individuals without any discrimination.

The Preamble secures to all citizens of India equality of status and opportunity. This provision
embraces three dimensions of equality—civic, political and economic.

14 Consider the following statements


1. Preamble is a source of power to legislature.
2. Preamble is part of the constitution
3. Preamble has been amended only once so far.

Which of the statements given above is/are correct?


A. 1 and 2 only
B. 3 only
C. 1 and 3 only
D. 2 and 3 only
1 3
Your Answer : D
5 27
2
38
Correct Answer : D
6
Answer Justification : -8
.c om
The Preamble is neither a source of power to legislature a il nor a prohibition upon the powers
of legislature. Hence Statement 1 is incorrect. g m
@
6 05is a part of the Constitution. Hence Statement 2 is
The Supreme Court held that the Preamble
5 12
correct.
m r7
o y-
The Preamble has been amended only once so far, in 1976, by the 42nd Constitutional Amendment
Act, which has addedRthree new words—Socialist, Secular and Integrity—to the Preamble. This
j oy
amendment was
u n held to be valid. Hence Statement 3 is correct.
i t
Mr
15 Consider the following statements
1. Being a sovereign state, India can acquire foreign territories according to the modes recognised by
international law.
2. Constitution authorizes the Parliament to form new states or alter the areas, boundaries or names
of the existing states without state’s consent.

Which of the statements given above is/are correct?


A. 1 only
B. 2 only
C. Both 1 and 2
D. Neither 1 nor 2

Your Answer :
Correct Answer : C

Answer Justification :

prelims.insightsonindia.com 10
© Insights Active Learning | All rights reserved - 156493. You may not reproduce, distribute or exploit the contents in any form without
written permission by copyright owner. Copyright infringers may face civil and criminal liability
Total Marks : 200
Online Prelims TEST - 18 (SUBJECT)
( InsightsIAS Mock Test Series for UPSC Preliminary Exam 2020 ) Mark Scored : 61.33

Both the statements are correct.

Being a sovereign state, India can acquire foreign territories according to the modes recognised by
international law, i.e., cession (following treaty, purchase, gift, lease or plebiscite), occupation
(hitherto unoccupied by a recognised ruler), conquest or subjugation.

Constitution authori-ses the Parliament to form new states or alter the areas, boundaries or names
of the existing states without state’s consent.

16 Consider the following statements


1. Bill to alter the boundaries of any state can be introduced in the Parliament only with the prior
recommendation of the President.
2. President has to refer the same bill to the state legistature concerned for expressing its views
within a specified period.
3. President is bound by the views of the state legislature.

1 3
Which of the statements given above is/are correct?
5 27
A. 3 only 2
B. 1 and 2 only 6 38
C. 2 and 3 only -8
o m
il.c
D. 2 only

Your Answer : gma


5 @
Correct Answer : B 0
126
5
Answer Justification :
r7
y -m
Article 3 authorises theoParliament to:
(a) form a new statey R
by separation of territory from any state or by uniting two or more states or
j o
parts of statesnor by uniting any territory to a part of any state,
r ituthe area of any state,
(b) increase
M the area of any state,
(c) diminish
(d) alter the boundaries of any state, and
(e) alter the name of any state.

However, Article 3 lays down two conditions in this regard: one, a bill contemplating the above
changes can be introduced in the Parliament only with the prior recommendation of the
President; Hence Statement 1 is correct.

Two, before recommending the bill, the President has to refer the same to the state legistature
concerned for expressing its views within a specified period. Hence Statement 2 is correct.

Further, the power of Parliament to form new states includes the power to form a new state or
union territory by uniting a part of any state or union territory to any other state or union territory.

The President (or Parliament) is not bound by the views of the state legislature and may
either accept or reject them, even if the views are received in time. Further, it is not necessary to
make a fresh reference to the state legislature every time an amendment to the bill is moved and

prelims.insightsonindia.com 11
© Insights Active Learning | All rights reserved - 156493. You may not reproduce, distribute or exploit the contents in any form without
written permission by copyright owner. Copyright infringers may face civil and criminal liability
Total Marks : 200
Online Prelims TEST - 18 (SUBJECT)
( InsightsIAS Mock Test Series for UPSC Preliminary Exam 2020 ) Mark Scored : 61.33

accepted in Parliament. Hence Statement 3 is incorrect.

17 Dhar Commission, JVP Committee, Fazl Ali Commission were constituted

A. To resolve inter-state water disputes.


B. To resolve minority issue after independence
C. To redraw the boundaries of the state
D. None of the above

Your Answer :
Correct Answer : C

Answer Justification :

Dhar Commission, JVP Committee, Fazl Ali Commission were constituted to redraw the boundaries
of the state. 3 1
7
52
82
18 Consider the following statements
8 63
-
1. Constitution contains neither any permanent nor any elaborate provisions regarding citizenship.
o m
2. If any foreign territory becomes a part of India, all persons
l .c of the territory shall become the citizens
of India. ai
@ gm
05
Which of the statements given above is/are correct?
6
A. 1 only
5 12
B. 2 only
m r7
C. Both 1 and 2 -
D. Neither 1 nor 2 oy

j oyR
Your Answer n :B
it u : A
Correct rAnswer
M
Answer Justification :

The Constitution deals with the citizenship from Articles 5 to 11 under Part II. However, it
contains neither any permanent nor any elaborate provisions in this regard. It only
identifies the persons who became citizens of India at its commencement (i.e., on January 26, 1950).
It does not deal with the problem of acquisition or loss of citizenship subsequent to its
commencement. Hence Statement 1 is correct.

It empowers the Parliament to enact a law to provide for such matters and any other matter relating
to citizenship.

If any foreign territory becomes a part of India, the Government of


India specifies the persons who among the people of the territory shall be the citizens of
India. Such persons become the citizens of India from the notified date. For example, when
Pondicherry became a part of India, the Government of India issued the Citizenship (Pondicherry)
Order, 1962, under the Citizenship Act, 1955. Hence Statement 2 is incorrect.

prelims.insightsonindia.com 12
© Insights Active Learning | All rights reserved - 156493. You may not reproduce, distribute or exploit the contents in any form without
written permission by copyright owner. Copyright infringers may face civil and criminal liability
Total Marks : 200
Online Prelims TEST - 18 (SUBJECT)
( InsightsIAS Mock Test Series for UPSC Preliminary Exam 2020 ) Mark Scored : 61.33

19 Consider the following statements


1. The Parliament can prescribe residence within a state or union territory as a condition for certain
employments or appointments.
2. The freedom of movement and residence of citizen in India is subjected to the protection of interests
of any schedule tribe.

Which of the statements given above is/are correct?


A. 1 only
B. 2 only
C. Both 1 and 2
D. Neither 1 nor 2

Your Answer : C
Correct Answer : C

Answer Justification :
1 3
5 27
Both the statements are correct. 2
6 38
The Parliament (under Article 16) can prescribe residence within- a8state or union territory as a

.c om or union territory, or local


condition for certain employments or appointments in that state
il
authority or other authority within that state or union territory

gma
The freedom of movement and residence (under
5 @ Article 19) is subjected to the protection of
interests of any schedule tribe. In other 6 0 the right of outsiders to enter, reside and settle in
words,
2
tribal areas is restricted.
7 51
- mr
oy
20 Consider the following statements
R
1. All fundamental o
j y are directly enforceable.
rights
2. Law can beu n by both parliament and state legislature to enforce fundamental rights
made
i t
Mr
Which of the statements given above is/are correct?
A. 1 only
B. 2 only
C. Both 1 and 2
D. Neither 1 nor 2

Your Answer :
Correct Answer : D

Answer Justification :

Both the statements are incorrect.

Most of fundamental rights are directly enforceable (self-executory) while a few of them
can be enforced on the basis of a law made for giving effect to them.

Such a law can be made only by the Parliament and not by state legislatures so that

prelims.insightsonindia.com 13
© Insights Active Learning | All rights reserved - 156493. You may not reproduce, distribute or exploit the contents in any form without
written permission by copyright owner. Copyright infringers may face civil and criminal liability
Total Marks : 200
Online Prelims TEST - 18 (SUBJECT)
( InsightsIAS Mock Test Series for UPSC Preliminary Exam 2020 ) Mark Scored : 61.33

uniformity throughout the country is maintained. (Article 35)

21 Consider the following statements


1. Article 11 declares that all laws that are inconsistent with or in derogation of any of the
fundamental rights shall be void.
2. The term ‘untouchability’ has been defined in Untouchability (Offences) Act, 1955
3. Freedom of movement of prostitutes can be restricted on the ground of public health and in the
interest of public morals.

Which of the statements given above is/are correct?


A. 2 and 3 only
B. 3 only
C. 1 and 3 only
D. 2 only

Your Answer : A 1 3
Correct Answer : B 5 27
2
6 38
Answer Justification :
-8
m
Article 13 declares that all laws that are inconsistent
i l .cowith or in derogation of any of the
fundamental rights shall be void. In other words, a it expressively provides for the doctrine of
gm
judicial review. This power has been conferred
5 @on the Supreme Court (Article 32) and the high
courts (Article 226) that can declare a law0unconstitutional and invalid on the ground of
6
12 Rights. Hence Statement 1 is incorrect.
contravention of any of the Fundamental
5
7
mr
- not been defined either in the Constitution or in the Act. However, the
The term ‘untouchability’ has
y
y Rothe
Mysore High Court held that the subject matter of Article 17 is not untouchability in its literal or
o
grammatical sense
j but ‘practice as it had developed historically in the country’. Hence
Statementu 2nis incorrect.
rit
M
The Supreme Court held that the freedom of movement of prostitutes can be restricted on
the ground of public health and in the interest of public morals. The Bombay High Court
validated the restrictions on the movement of persons affected by AIDS. Hence Statement 3 is
correct.

22 Which of the following rights is/are declared by Supreme Court as a part of Article 21
1. Right to privacy.
2. Right against handcuffing
3. Right against public hanging.
4. Right to marry

Which of the statements given above is/are correct?


A. 1 and 2 only
B. 1 and 3 only
C. 4 only
D. 1, 2, 3 and 4

prelims.insightsonindia.com 14
© Insights Active Learning | All rights reserved - 156493. You may not reproduce, distribute or exploit the contents in any form without
written permission by copyright owner. Copyright infringers may face civil and criminal liability
Total Marks : 200
Online Prelims TEST - 18 (SUBJECT)
( InsightsIAS Mock Test Series for UPSC Preliminary Exam 2020 ) Mark Scored : 61.33

Your Answer :
Correct Answer : D

Answer Justification :

The Supreme Court has reaffirmed its judgement in the Menaka case in the subsequent cases. It has
declared the following rights as part of Article 21:
(a) Right to live with human dignity.
(b) Right to decent environment including pollution free water and air and protection against
hazardous industries.
(c) Right to livelihood.
(d) Right to privacy.
(e) Right to shelter.
(f) Right to health.
(g) Right to free education up to 14 years of age.
(h) Right to free legal aid.
(i) Right against solitary confinement.
7 13
(j) Right to speedy trial.
2 52
(k) Right against handcuffing.
6 38
(l) Right against inhuman treatment.
-8
om
(m) Right against delayed execution.
(n) Right to travel abroad.
il .c
a
gm
(o) Right against bonded labour.
(p) Right against custodial harassment. @
(q) Right to emergency medical aid. 6 05
(r) Right to timely medical treatment1in 2government hospital.
5
(s) Right not to be driven out ofra7state.
(t) Right to fair trial.
y -m
(u) Right of prisonerR toohave necessities of life.
y
jo to be treated with decency and dignity.
(v) Right of women
n
r i tu public hanging.
(w) Right against
M to hearing.
(x) Right
(y) Right to information.
(z) Right to reputation
(ab) Right to marry

23 Consider the following statements


1. In case of the enforcement of Fundamental Rights, the jurisdiction of the Supreme Court is original
but not exclusive.
2. No tax shall be levied or collected except by authority of law
3. The Supreme Court can issue writs only for the enforcement of fundamental rights

Which of the statements given above is/are correct?


A. 1 and 2 only
B. 3 only
C. 1 and 3 only
D. 1, 2 and 3

prelims.insightsonindia.com 15
© Insights Active Learning | All rights reserved - 156493. You may not reproduce, distribute or exploit the contents in any form without
written permission by copyright owner. Copyright infringers may face civil and criminal liability
Total Marks : 200
Online Prelims TEST - 18 (SUBJECT)
( InsightsIAS Mock Test Series for UPSC Preliminary Exam 2020 ) Mark Scored : 61.33

Your Answer : D
Correct Answer : D

Answer Justification :

All the statements given above are correct.

In case of the enforcement of Fundamental Rights, the jurisdiction of the Supreme Court is original
but not exclusive. It is concurrent with the jurisdiction of the high court under Article 226. It vests
original powers in the high court to issue directions, orders and writs of all kinds for the
enforcement
of the Fundamental Rights.

These rights are known as constitutional rights or legal rights or non -fundamental rights. They are:

1. No tax shall be levied or collected except by authority of law (Article 265 in Part3XII).
2. No person shall be deprived of his property save by authority of law (Article 7 1 in Part XII).
300-A
5 2
82
8 63
The Supreme Court can issue writs only for the enforcement of fundamental rights whereas a high
-
court can issue writs not only for the enforcement of Fundamental Rights but also for any other
purpose. o m
il .c
m a
24 Consider the following statements @g
6 05 of Instructions’ enumerated in the Government of
1. Directive Principles resemble the ‘Instrument
India Act of 1935. 5 12
r7 in examining and determining the constitutional validity of a
2. Directive Principles help the courts
m
-
oy
law.

y R
j o
Which of the statements
n given above is/are correct?
u
rit
A. 1 only
B.M2 only
C. Both 1 and 2
D. Neither 1 nor 2

Your Answer : A
Correct Answer : C

Answer Justification :

Both the statements are correct.

The Directive Principles resemble the ‘Instrument of Instructions’ enumerated in the Government of
India Act of 1935. In the words of Dr B R Ambedkar, ‘the Directive Principles are like the
instrument of instructions, which were issued to the Governor-General and to the Governors of the
colonies of India by the British Government under the Government of India Act of 1935.

The Directive Principles, though non-justiciable in nature, help the courts in examining and

prelims.insightsonindia.com 16
© Insights Active Learning | All rights reserved - 156493. You may not reproduce, distribute or exploit the contents in any form without
written permission by copyright owner. Copyright infringers may face civil and criminal liability
Total Marks : 200
Online Prelims TEST - 18 (SUBJECT)
( InsightsIAS Mock Test Series for UPSC Preliminary Exam 2020 ) Mark Scored : 61.33

determining the constitutional validity of a law.

25 The framers of the Constitution made the Directive Principles non-justiciable. What were the reasons
for it?
1. The country did not possess sufficient financial resources to implement them
2. It might divide the country into different parts.
3. The presence of vast diversity and backwardness in the country.

Select the correct answer using the code given below


A. 1 and 2 only
B. 3 only
C. 1 and 3 only
D. 2 and 3 only

Your Answer : C
Correct Answer : C 1 3
527
2
Answer Justification :
38
- 86
The framers of the Constitution made the Directive Principles non-justiciable and legally non-
m
enforceable because: coil.
a
1. The country did not possess sufficient financial
@ gmresources to implement them.
0 5
2. The presence of vast diversity and backwardness in the country would stand in the way of their
6
implementation.
5 12State with its many preoccupations might be crushed under
the burden unless it was freem
r7
3. The newly born independent Indian
to decide the order, the time, the place and the mode of fulfilling
them. Hence Statement o y 2- is incorrect.
j oyR
u n
i
26 Consider the
r t following statements regarding Fundamental duties
1. TheyMextend to all persons whether citizens or foreigners
2. Some of them are moral duties while others are civic duties.

Which of the statements given above is/are correct?


A. 1 only
B. 2 only
C. Both 1 and 2
D. Neither 1 nor 2

Your Answer : B
Correct Answer : B

Answer Justification :

Some of fundamental duties are moral duties while others are civic duties. For instance,
cherishing noble ideals of freedom struggle is a moral precept and respecting the Constitution,
National Flag and National Anthem is a civic duty. Hence statement 2 is correct.

prelims.insightsonindia.com 17
© Insights Active Learning | All rights reserved - 156493. You may not reproduce, distribute or exploit the contents in any form without
written permission by copyright owner. Copyright infringers may face civil and criminal liability
Total Marks : 200
Online Prelims TEST - 18 (SUBJECT)
( InsightsIAS Mock Test Series for UPSC Preliminary Exam 2020 ) Mark Scored : 61.33

Unlike some of the Fundamental Rights which extend to all persons whether citizens or foreigners,
the Fundamental Duties are confined to citizens only and do not extend to foreigners. Hence
statement 1 is incorrect.

27 The ideals of Liberty, equality and fraternity in Indian constitution have been borrowed from

A. American Constitution
B. British Constitution
C. France Constitution
D. Canadian Constitution

Your Answer : C
Correct Answer : C

Answer Justification :

7 13
52 France
Ideals of Liberty, equality and fraternity in Indian constitution is borrowed from
2
Constitution.
6 38
-8
28 Consider the following statements
.c om
1. Right against phone tapping is implicit under Article a il
21
2. Freedom to form association or unions is guaranteed
@ gm by Article 19
5
0correct?
6
12
Which of the statements given above is/are
5
A. 1 only
B. 2 only m r7
C. Both 1 and 2 o y-
D. Neither 1 nor
j oy 2R
i t un
Mr
Your Answer :C
Correct Answer : B

Answer Justification :

The Right against Phone tapping is comes under Right to Freedom of speech and
expression (Art 19). Hence Statement 1 is incorrect.

Freedom to form association or unions is guaranteed by Article 19. Hence Statement 2 is correct.

29 Consider the following statements


1. Amendment of certain provisions of the Constitution by a simple majority is not deemed as
constitutional amendment.
2. The majority of the provisions in the Constitution need to be amended by a special majority of the
Parliament
3. National emergency can be imposed by the President when the security of India is threatened by
war or external aggression only.

prelims.insightsonindia.com 18
© Insights Active Learning | All rights reserved - 156493. You may not reproduce, distribute or exploit the contents in any form without
written permission by copyright owner. Copyright infringers may face civil and criminal liability
Total Marks : 200
Online Prelims TEST - 18 (SUBJECT)
( InsightsIAS Mock Test Series for UPSC Preliminary Exam 2020 ) Mark Scored : 61.33

Which of the statements given above is/are correct?


A. 1 and 2 only
B. 2 only
C. 3 only
D. 1, 2 and 3

Your Answer : B
Correct Answer : A

Answer Justification :

Some other articles provide for the amendment of certain provisions of the Constitution by
a simple majority of Parliament, that is, a majority of the members of each House present and
voting (similar to the ordinary legislative process). Notably, these amendments are not deemed to
be amendments of the Constitution for the purposes of Article 368.

The majority of the provisions in the Constitution need to be amended by1 a3special
7
52 of the total
majority of the Parliament, that is, a majority (that is, more than 50 per cent)
2
membership of each House and a majority of two-thirds of the members8of each House present and
3
voting. 86 -
National emergency can be imposed on the basis of.c om
armed rebellion also. Hence Statement
3 is incorrect. ail
@ gm
0 5
26basic structure of Indian Constitution?
30 Which of the following elements is/are the
1
1. Harmony between Union and State
r 75
2. Powers of the Supreme Courtm
y - under Articles 32
o
3. Unity and integrity of the nation

j oyR
Which of the n statements given above is/are correct?
A. 3 r i
onlyt u
M
B. 2 only
C. 1 and 3 only
D. 2 and 3 only

Your Answer : D
Correct Answer : D

Answer Justification :

From the various judgements, the following have emerged as ‘basic features’ of the Constitution or
elements / components / ingredients of the ‘basic structure’ of the constitution:

1. Supremacy of the Constitution


2. Sovereign, democratic and republican nature of the Indian polity
3. Secular character of the Constitution
4. Separation of powers between the legislature, the executive and the judiciary

prelims.insightsonindia.com 19
© Insights Active Learning | All rights reserved - 156493. You may not reproduce, distribute or exploit the contents in any form without
written permission by copyright owner. Copyright infringers may face civil and criminal liability
Total Marks : 200
Online Prelims TEST - 18 (SUBJECT)
( InsightsIAS Mock Test Series for UPSC Preliminary Exam 2020 ) Mark Scored : 61.33

5. Federal character of the Constitution


6. Unity and integrity of the nation
7. Welfare state (socio-economic justice)
8. Judicial review
9. Freedom and dignity of the individual
10. Parliamentary system
11. Rule of law
12. Harmony and balance between Fundamental Rights and Directive Principles
13. Principle of equality

14. Free and fair elections


15. Independence of Judiciary
16. Limited power of Parliament to amend the Constitution
17. Effective access to justice
18. Principle of reasonableness
19. Powers of the Supreme Court under Articles 32.

7 13
52
Harmony between Union and State is not a part of basic structure of constitution.
2
6 38
31 Consider the following statements -8
c om body set up by the Government of
1. The Telecom Regulatory Authority of India (TRAI) is a statutory
.
a
India under Telecom Regulatory Authority of India Act, il1997.
@ gm
2. TRAI regulate telecom services, including fixation/revision of tariffs for telecom services which were
earlier vested in the Central Government. 5
0
6and
3. Telecommunications Dispute Settlement
1 2 Appellate Tribunal adjudicates any dispute between a
5
licensor and a licensee and hears7and dispose of appeals against any direction, decision or order of
Central government.
- mr
R oy
j oy
Which of the statements given above is/are correct?
A. 1, 2 andn3
B. 3 r it u
only
C.M1 and 3 only
D. 1 and 2 only

Your Answer : A
Correct Answer : D

Answer Justification :

The Telecom Regulatory Authority of India (TRAI) is a statutory body set up by the
Government of India under Telecom Regulatory Authority of India Act, 1997. Hence
Statement 1 is correct.

TRAI regulate telecom services, including fixation/revision of tariffs for telecom services
which were earlier vested in the Central Government. Hence Statement 2 is correct.

The TRAI Act was amended by an ordinance, effective from 24 January 2000, establishing a
Telecommunications Dispute Settlement and Appellate Tribunal (TDSAT) to take over the

prelims.insightsonindia.com 20
© Insights Active Learning | All rights reserved - 156493. You may not reproduce, distribute or exploit the contents in any form without
written permission by copyright owner. Copyright infringers may face civil and criminal liability
Total Marks : 200
Online Prelims TEST - 18 (SUBJECT)
( InsightsIAS Mock Test Series for UPSC Preliminary Exam 2020 ) Mark Scored : 61.33

adjudicatory and disputes functions from TRAI.

TDSAT was set up to adjudicate any dispute between a licensor and a licensee, between two or
more service providers, between a service provider and a group of consumers, and to hear and
dispose of appeals against any direction, decision or order of TRAI. Hence Statement 3 is
incorrect.

https://main.trai.gov.in/about-us/history

32 Which of the following departments comes under the direct control Prime Minister’s Office (PMO)?
1. Department of Defence
2. Department of Space
3. Department of Atomic Energy

Select the correct answer using the code given below


A. 3 only
1 3
B. 2 only
5 27
2
38
C. 2 and 3 only
D. 1, 2 and 3 6
-8
o m
il.c
Your Answer :

ma
Correct Answer : C

@ g
Answer Justification : 0 5
1 26
75 Energy comes directly under Prime Minister.
Department of Space and Atomic

- mr
Department of defence
o y comes under Ministry of Defence, which is in turn headed by Defence
Minister.
j oyR
i t un
Mr following statements
33 Consider the
1. A No-Confidence Motion should state the reasons for its adoption in the Lok Sabha.
2. If Censure motion is passed in the Lok Sabha, the council of ministers need not resign from the
office.
3. Adjournment Motion is introduced in the Parliament to draw attention of the House to a definite
matter of urgent public importance and needs the support of 100 members to be admitted.

Which of the statements given above is/are correct?


A. 2 and 3 only
B. 1 and 3 only
C. 1, 2 and 3 only
D. 2 only

Your Answer : D
Correct Answer : D

Answer Justification :

prelims.insightsonindia.com 21
© Insights Active Learning | All rights reserved - 156493. You may not reproduce, distribute or exploit the contents in any form without
written permission by copyright owner. Copyright infringers may face civil and criminal liability
Total Marks : 200
Online Prelims TEST - 18 (SUBJECT)
( InsightsIAS Mock Test Series for UPSC Preliminary Exam 2020 ) Mark Scored : 61.33

A No-Confidence Motion need not state the reasons for its adoption in the Lok Sabha. Hence
Statement 1 is incorrect.

Adjournment Motion is introduced in the Parliament to draw attention of the House to a


definite matter of urgent public importance and needs the support of 50 members to be admitted.
Hence Statement 3 is incorrect.

34 Consider the following statements regarding Lok Sabha


1. Lok Sabha has a normal term of 5 years, after which it automatically dissolves.
2. Revocation of National Emergency is the exclusive power of Lok Sabha

Which of the statements given above is/are correct?


A. 1 only
B. 2 only
C. Both 1 and 2
D. Neither 1 nor 2
1 3
5 27
2
Your Answer : A
6 38
-8
Correct Answer : A

om
il.c
Answer Justification :
a
gm it automatically dissolves. Hence, Statement
Lok Sabha has a normal term of 5 years, after which
@
1 is correct. 05
1 26
Revocation of National Emergency
r 75is not the exclusive power of Lok Sabha. Even President of
India can revoke the nationalm emergency. Hence Statement 2 is incorrect.
-
R oy
35 With reference ofjo
y
the office of the Lok Sabha Speaker, consider the following statements:
i t
1. He/ She hold unthe office during the pleasure of the President.
Mrneed not be a member of the House at the time of his/her election but has to become a
2. He/ She
member of the House within six months from the date of his/her election.
3. If he/she intends to resign, the letter of his/her resignation has to be addressed to the Deputy
Speaker.

Which of the statements given above is/are correct?


A. 3 only
B. 1 and 2 only
C. 2 and 3 only
D. None

Your Answer : A
Correct Answer : A

Answer Justification :

Each House of Parliament has its own presiding officer. There is a Speaker and a Deputy Speaker

prelims.insightsonindia.com 22
© Insights Active Learning | All rights reserved - 156493. You may not reproduce, distribute or exploit the contents in any form without
written permission by copyright owner. Copyright infringers may face civil and criminal liability
Total Marks : 200
Online Prelims TEST - 18 (SUBJECT)
( InsightsIAS Mock Test Series for UPSC Preliminary Exam 2020 ) Mark Scored : 61.33

for the Lok Sabha and a Chairman and a Deputy Chairman for the Rajya Sabha. A panel of
chairpersons for the Lok Sabha and a panel of vice-chairpersons for the Rajya Sabha is also
appointed.

The Speaker is elected by the Lok Sabha from amongst its members. That means he should have
been already a member of Loksabha. Hence Statement 2 is incorrect.

He is provided with a security of tenure. He can be removed only by a resolution passed by


the Lok Sabha by an absolute majority (ie, a majority of the total members of the House)
Hence Statement 1 is incorrect.

He can resign by writing to the Deputy Speaker. Hence Statement 3 is correct.

36 Consider the following statements


1. Council of Ministers are collectively responsible to the Parliament
2. Resignation or death of an incumbent Chief Minister dissolves the Council of Ministers.
1 3
5 27
Which of the statements given above is/are correct? 2
A. 1 only 6 38
B. 2 only -8
C. Both 1 and 2 o m
il.c
ma
D. Neither 1 nor 2

@ g
Your Answer : B
0 5
Correct Answer : B
126
5
Answer Justification : m r7
-
oy
j oy R are collectively responsible to the Loka Sabha. Chairman of Rajya Sabha
Council of Ministers
n over a joint sitting of the two houses of the Parliament in any circumstance. Hence
cannot preside
u
Statementt
ri is incorrect.
1
M
Resignation or death of an incumbent Chief Minister dissolves the Council of Ministers. Hence
Statement 2 is correct.

37 Consider the following statements regarding Zonal Council


1. Zonal Councils are established by the States Reorganization Act of 1956.
2. Prime Minister is the common chairman of the five zonal councils.

Which of the statements given above is/are correct?


A. 1 only
B. 2 only
C. Both 1 and 2
D. Neither 1 nor 2

Your Answer : A
Correct Answer : A

prelims.insightsonindia.com 23
© Insights Active Learning | All rights reserved - 156493. You may not reproduce, distribute or exploit the contents in any form without
written permission by copyright owner. Copyright infringers may face civil and criminal liability
Total Marks : 200
Online Prelims TEST - 18 (SUBJECT)
( InsightsIAS Mock Test Series for UPSC Preliminary Exam 2020 ) Mark Scored : 61.33

Answer Justification :

The Zonal Councils are the statutory bodies. They are established by an Act of
the Parliament, that is, States Reorganisation Act of 1956. The act divided
the country into five zones (Northern, Central, Eastern, Western and
Southern) and provided a zonal council for each zone. Hence Statement 1 is correct.

The home minister of Central government is the common chairman of the


five zonal councils. Each chief minister acts as a vice-chairman of the
council by rotation, holding office for a period of one year at a time. Hence Statement 2 is
incorrect.

38 Consider the following statements


1. The states are given representation in the Rajya Sabha on the basis of population.
2. The Comptroller and Auditor-General of India audits the accounts of Central government only.

1 3
Which of the statements given above is/are correct?
5 27
2
38
A. 1 only
B. 2 only 6
C. Both 1 and 2 -8
o m
il.c
D. Neither 1 nor 2

Your Answer : A gma


5 @
Correct Answer : A 0
126
5
Answer Justification :
r7
-m
R oy
The states are given representation in the Rajya Sabha on the basis of population. Hence, the
membership varies
j oy from 1 to 31. In US, on the other hand, the principle of equality of
n states in the Upper House is fully recognised. Hence Statement 1 is correct.
representation of
rit u
M
The Comptroller and Auditor-General of India audits the accounts of not only the Central
government but also those of the states. But, his appointment and removal is done by the president
without consulting the states. Hence Statement 2 is incorrect.

39 Consider the following statements


1. The state legislature has exclusive powers in all circumstances to make laws with respect to any of
the matters enumerated in the State List.
2. Rajya Sabha can declare that Parliament could make laws on a matter in the State List.

Which of the statements given above is/are correct?


A. 1 only
B. 2 only
C. Both 1 and 2
D. Neither 1 nor 2

Your Answer : B

prelims.insightsonindia.com 24
© Insights Active Learning | All rights reserved - 156493. You may not reproduce, distribute or exploit the contents in any form without
written permission by copyright owner. Copyright infringers may face civil and criminal liability
Total Marks : 200
Online Prelims TEST - 18 (SUBJECT)
( InsightsIAS Mock Test Series for UPSC Preliminary Exam 2020 ) Mark Scored : 61.33

Correct Answer : B

Answer Justification :

If the Rajya The state legislature has “in normal circumstances” exclusive powers to make laws with
respect to any of the matters enumerated in the State List. This has at present 61 subjects
(originally 66 subjects) like public order, police, public health and sanitation, agriculture, prisons,
local government, fisheries, markets, theaters, gambling and so on. Hence Statement 1 is
incorrect.

Rajya Sabha declares that it is necessary in the national interest that Parliament should make laws
on a matter in the State List, then the Parliament becomes competent to make laws on that matter.
Hence Statement 2 is correct.

40 Which of the following matter Centre is empowered to give directions to the states with regard to the
exercise of their executive power?
1. the construction and maintenance of means of communication by the state. 7 13
2. the measures to be taken for the protection of the railways within the state2 52
6
3. the provision of adequate facilities for instruction in the mother-tongue38
at the primary stage of
education to children belonging to linguistic minority groups - 8
o m
Which of the statements given above is/are correct? a il.c
A. 1 only
@ gm
B. 2 only
6 05
C. 2 and 3 only
5 12
D. 1, 2 and 3 7 r
m
y-
Ro
Your Answer : D
y
Correct Answer : D
u njo
it
AnswerrJustification :
M
All the statements given above are correct.

Centre’s Directions to the States

In addition to the above two cases, the Centre is empowered to give directions to the states with
regard to the exercise of their executive power in the following matters:
(i) the construction and maintenance of means of communication (declared to be of national or
military importance) by the state;
(ii) the measures to be taken for the protection of the railways within the state;
(iii) the provision of adequate facilities for instruction in the mother-tongue at the primary stage of
education to children belonging to linguistic minority groups in the state; and
(iv) the drawing up and execution of the specified schemes for the welfare of the Scheduled Tribes
in the state.

prelims.insightsonindia.com 25
© Insights Active Learning | All rights reserved - 156493. You may not reproduce, distribute or exploit the contents in any form without
written permission by copyright owner. Copyright infringers may face civil and criminal liability
Total Marks : 200
Online Prelims TEST - 18 (SUBJECT)
( InsightsIAS Mock Test Series for UPSC Preliminary Exam 2020 ) Mark Scored : 61.33

41 Section 8 of the Representation of the People (RP) Act, 1951 that is frequently in news concerns

A. Disqualification on conviction for long sentences


B. Division of seats in the Upper house
C. Horse trading in Council elections
D. Paid advertisements for election campaign

Your Answer : A
Correct Answer : A

Answer Justification :

Section 8 disqualifies a person convicted with a sentence of two years or more from contesting
elections. But those under trial continued to be eligible to contest elections. The Lily Thomas case
(2013), however, ended this unfair advantage.

13elections only
Currently, under the Representation of Peoples (RP) Act, lawmakers cannot contest
7
after their conviction in a criminal case. 52 2
8
8 63
-
42 Consider the following statements regarding the Election of the President
m
i l coPresident, must secure a fixed quota of
1. A candidate, in order to be declared elected to the office.of
votes.
m a
g
@challenged on the ground that the Electoral College
2. The election of a person as President cannot be
was incomplete 0 5
26 election of the President are inquired into and decided
3. All doubts and disputes in connection1with
by Election commission of Indiar7
5
y -m
Ro given above is/are correct?
Which of the statements
y
njo
A. 1 and 2 only

it
B. 3 only u
C. 2 r
only
M
D. 2 and 3 only

Your Answer : D
Correct Answer : A

Answer Justification :

A candidate, in order to be declared elected to the office of President, must secure a fixed quota of
votes. The quota of votes is
determined by dividing the total number of valid votes polled by the number of candidates to be
elected (here only one candidate is to be elected as President) plus one and adding one to the
quotient Hence Statement 1 is correct

The election of a person as President cannot be challenged on the ground that the Electoral
College was incomplete. Hence Statement 2 is correct

All doubts and disputes in connection with election of the President are inquired into and decided
prelims.insightsonindia.com 26
© Insights Active Learning | All rights reserved - 156493. You may not reproduce, distribute or exploit the contents in any form without
written permission by copyright owner. Copyright infringers may face civil and criminal liability
Total Marks : 200
Online Prelims TEST - 18 (SUBJECT)
( InsightsIAS Mock Test Series for UPSC Preliminary Exam 2020 ) Mark Scored : 61.33

by the Supreme Court whose decision is final. Hence, statement 3 is incorrect.

43 Consider the following statements


1. Absolute veto refers to withholding of assent to the bill passed by the legislature.
2. Pocket veto refers to taking no action on the bill passed by the legislature.

Which of the statements given above is/are correct?


A. 1 only
B. 2 only
C. Both 1 and 2
D. Neither 1 nor 2

Your Answer : C
Correct Answer : C

Answer Justification : 1 3
527
2
Both the statements are correct.
38
86
The veto power enjoyed by the executive in modern states can be- classified into the following four
m
types: coil.
a
mbill passed by the legislature.
1. Absolute veto that is, withholding of assent togthe
5 @
6 0by
2. Qualified veto, which can be overridden
1 2 the legislature with a higher majority.
5
7over
3. Suspensive veto, which can be
m r ridden by the legislature with an ordinary majority.
-
4. Pocket veto that is,R oy no action on the bill passed by the legislature.
taking
j oy
u n
i t
Mr
44 Consider the following statements
1. President of India is not a member of either House of Parliament.
2. The Fourth Schedule of the Constitution deals with the allocation of seats in the Rajya Sabha to the
states only
3. The Constitution has empowered the Parliament to prescribe the manner of choosing the
representatives of the union territories in the Lok Sabha.

Which of the statements given above is/are correct?


A. 3 only
B. 2 only
C. 2 and 3 only
D. 1 and 3 only

Your Answer :
Correct Answer : D

Answer Justification :

prelims.insightsonindia.com 27
© Insights Active Learning | All rights reserved - 156493. You may not reproduce, distribute or exploit the contents in any form without
written permission by copyright owner. Copyright infringers may face civil and criminal liability
Total Marks : 200
Online Prelims TEST - 18 (SUBJECT)
( InsightsIAS Mock Test Series for UPSC Preliminary Exam 2020 ) Mark Scored : 61.33

Though the President of India is not a member of either House of Parliament and does not sit in the
Parliament to attend its meetings, he is an integral part of the Parliament. This is because a bill
passed by both the Houses of Parliament cannot become law without the President’s assent. Hence
Statement 1 is correct

The Fourth Schedule of the Constitution deals with the allocation of seats in the Rajya
Sabha to the states and union territories Hence Statement 2 is incorrect

The Constitution has empowered the Parliament to prescribe the manner of choosing the
representatives of the union territories in the Lok Sabha.
Accordingly, the Parliament has enacted the Union Territories (Direct Election to the House of the
People) Act, 1965, by which the members of Lok Sabha from the union territories are also chosen by
direct election. Hence Statement 3 is correct

45 Consider the following statements


1. Each state is allotted a number of seats in the Lok Sabha in such a manner that the3ratio between
that number and its population is the same for all states 2 71
2. Each state is divided into territorial constituencies in such a manner that 2 5 ratio between the
the
8
population of each constituency and the number of seats allotted to it
8 63is the same throughout the
state -
m
i l .co
ma
Which of the statements given above is/are not correct?
A. 1 only g
B. 2 only 0 5@
C. Both 1 and 2
1 26
D. Neither 1 nor 2 r 75
y -m
Your Answer : C
Correct Answer o y
:D
Ro
u nj
it
AnswerrJustification :
M
Both the statements are correct.

For the purpose of holding direct elections to the Lok Sabha, each state is divided into territorial
constituencies. In this respect, the Constitution makes the following two provisions:

1. Each state is allotted a number of seats in the Lok Sabha in such a manner that the ratio between
that number and its population is the same for all states. This provision does not apply to a state
having a population of less than six millions.

2. Each state is divided into territorial constituencies in such a manner that the ratio between the
population of each constituency and the number of seats allotted to it is the same throughout the
state

46 Consider the following statements


1. The Constitution has not fixed the term of office of members of the Rajya Sabha

prelims.insightsonindia.com 28
© Insights Active Learning | All rights reserved - 156493. You may not reproduce, distribute or exploit the contents in any form without
written permission by copyright owner. Copyright infringers may face civil and criminal liability
Total Marks : 200
Online Prelims TEST - 18 (SUBJECT)
( InsightsIAS Mock Test Series for UPSC Preliminary Exam 2020 ) Mark Scored : 61.33

2. President is authorised to dissolve the Lok Sabha at any time even before the completion of five
years

Which of the statements given above is/are correct?


A. 1 only
B. 2 only
C. Both 1 and 2
D. Neither 1 nor 2

Your Answer : B
Correct Answer : C

Answer Justification :

Both the statements are correct.

13 left it to the
The Constitution has not fixed the term of office of members of the Rajya Sabha and
7
2 (1951) provided
Parliament. Accordingly, the Parliament in the Representation of the People5Act
8
that the term of office of a member of the Rajya Sabha shall be six years. 2
6 3
-8
The President is authorised to dissolve the Lok Sabha at any time even before the completion of five
m
years and this cannot be challenged in a court of law. .co
a il
m
@g
47 Consider the following statements
6 05
1 2
1. Speaker appoints the chairman of all the parliamentary committees of the Lok Sabha and supervises
5
their functioning.
2. Speaker salaries are chargedm
r7
on the Consolidated Fund of India and thus are not subject to the
-
oy
annual vote of Parliament.

j oyR
Which of the n statements given above is/are correct?
A. 1 r i
onlyt u
M
B. 2 only
C. Both 1 and 2
D. Neither 1 nor 2

Your Answer : C
Correct Answer : C

Answer Justification :

Both the statements are correct.

Speaker appoints the chairman of all the parliamentary committees of the Lok Sabha and
supervises their functioning. He himself is the chairman of the Business Advisory Committee, the
Rules Committee and the General Purpose Committee.

Speaker salaries and allowances are fixed by Parliament. They are charged on the
Consolidated Fund of India and thus are not subject to the annual vote of Parliament.

prelims.insightsonindia.com 29
© Insights Active Learning | All rights reserved - 156493. You may not reproduce, distribute or exploit the contents in any form without
written permission by copyright owner. Copyright infringers may face civil and criminal liability
Total Marks : 200
Online Prelims TEST - 18 (SUBJECT)
( InsightsIAS Mock Test Series for UPSC Preliminary Exam 2020 ) Mark Scored : 61.33

Speaker salaries and allowances are fixed by Parliament. They are charged on the
Consolidated Fund of India and thus are not subject to the annual vote of Parliament.

48 Consider the following statements


1. After the declaration of results in all constituencies President will constitute the new Lok Sabha.
2. The Returning Officer is responsible for the counting of votes and declaration of result of an
election.

Which of the statements given above is/are correct?


A. 1 only
B. 2 only
C. Both 1 and 2
D. Neither 1 nor 2

Your Answer :
Correct Answer : B 1 3
527
2
Answer Justification :
38
- 86
After the declaration of results in all constituencies, which authority will constitute the
new Lok Sabha – President or the Election Commission? .c om
Ans. Election Commission of India (ECI) a il
@ gm
5after the results of all Parliamentary constituencies are
60
According to Sec. 73 of the R. P. Act, 1951,
declared, the Election Commission will2constitute the new Lok Sabha by notifying in the official
5 1
gazette, the names of the elected7members.
- mr
49 Which among the followingR oy can authorize the Parliament to create new All-India Services common to
both the Centre andj oystates?
i t un
A.M
r
President of India
B. Speaker, Lok Sabha
C. Rajya Sabha
D. Inter-State Council

Your Answer : C
Correct Answer : C

Answer Justification :

Due to its federal character, the Rajya Sabha has been given two exclusive or special powers that
are not enjoyed by the Lok Sabha:

1. It can authorise the Parliament to make a law on a subject enumerated in the State List (Article
249).

2. It can authorise the Parliament to create new All-India Services common to both the Centre and

prelims.insightsonindia.com 30
© Insights Active Learning | All rights reserved - 156493. You may not reproduce, distribute or exploit the contents in any form without
written permission by copyright owner. Copyright infringers may face civil and criminal liability
Total Marks : 200
Online Prelims TEST - 18 (SUBJECT)
( InsightsIAS Mock Test Series for UPSC Preliminary Exam 2020 ) Mark Scored : 61.33

states (Article 312).

50 Consider the following statements


1. A bill pending in the Rajya Sabha but not passed by the Lok Sabha does not lapse after the
dissolution of Lokasabha.
2. A bill passed by the Lok Sabha but pending in the Rajya Sabha does not lapse after the dissolution
of Lokasabha.
3. A bill pending in the Lok Sabha does not lapses after the dissolution of Lokasabha.

Which of the statements given above is/are correct?


A. 1 only
B. 1 and 3 only
C. 2 and 3 only
D. 2 only

Your Answer : 1 3
Correct Answer : A 5 27
2
6 38
Answer Justification :
-8
m
i l .co of LokaSabha is as follows:
The position with respect to lapsing of bills after the dissolution

m a
1. A bill pending in the Lok Sabha lapses (whether originating in the Lok Sabha or transmitted to it
by the Rajya Sabha). g
2. A bill passed by the Lok Sabha but pending0 5@in the Rajya Sabha lapses.
6
3. A bill not passed by the two Houses
5 12due to disagreement and if the president has notified the
holding of a joint sitting before r 7 dissolution of Lok Sabha, does not lapse.
the
-
4. A bill pending in the Rajya m Sabha but not passed by the Lok Sabha does not lapse.
y
5. A bill passed by bothoHouses but pending assent of the president does not lapse.
R Houses but returned by the president for reconsideration of Houses does
yboth
6. A bill passed by
j o
not lapse. un
rit
M
51 Which of the following Constitutional Amendment gave the constitutional status and protection to co-
operative societies?

A. 28th Constitutional Amendment Act, 1972


B. 42nd Constitutional Amendment Act, 1976
C. 64th Constitutional Amendment Act, 1990
D. 97th Constitutional Amendment Act, 2011

Your Answer : D
Correct Answer : D

Answer Justification :

A co-operative society is a type of volunteer association. Article 19 states that the Right to form co-
operative societies is a fundamental right and Article 43-B provides for the promotion of co-
operative societies (DPSP). The 97thConstitutional Amendment Act of 2011 gave a constitutional

prelims.insightsonindia.com 31
© Insights Active Learning | All rights reserved - 156493. You may not reproduce, distribute or exploit the contents in any form without
written permission by copyright owner. Copyright infringers may face civil and criminal liability
Total Marks : 200
Online Prelims TEST - 18 (SUBJECT)
( InsightsIAS Mock Test Series for UPSC Preliminary Exam 2020 ) Mark Scored : 61.33

status and protection to co-operative societies. Hence, option (d) is correct.

52 With reference to administration of Union Territories, consider the following statements


1. Every union territory is administered by the President acting through an administrator appointed by
him.
2. An administrator of a union territory is an agent of the President and head of state like a governor.

Which of the statements given above is/are correct?


A. 1 only
B. 2 only
C. Both 1 and 2
D. Neither 1 nor 2

Your Answer : C
Correct Answer : A
1 3
Answer Justification : 527
2
Under Article 1 of the Constitution, the territory of India comprises8three
38
6 categories of territories:
-
om states, seven union territories
(a) territories of the states; (b) union territories; and (c) territories that may be acquired by the
.c
Government of India at any time. At present, there are twenty-nine
il
m a
and no acquired territories. Every union territory is administered by the President acting through
g
an administrator appointed by him. Hence, statement 1 is correct.
@
5
60
An administrator of a union territory1is2an agent of the President and not head of state like a
r 75 the designation of an administrator; it may be Lieutenant
governor. The President can specify
- m or Administrator. Hence, statement 2 is incorrect.
Governor or Chief Commissioner

R oy
tojo
53 With reference n
y
Administration of Tribal Areas, consider the following statements.
r
1. The Sixth tu
iSchedule of the constitution provides special provisions for the administration of the
M
tribal areas
2. The autonomous district has a district council consisting of not more than fifty members.
3. The governor can increase, decrease, re-organize or alter the boundary of these districts.

Which of the statements given above is/are correct?


A. 2 only
B. 1 and 3 only
C. 2 and 3 only
D. 1, 2 and 3

Your Answer : C
Correct Answer : B

Answer Justification :

The Sixth Schedule of the constitution provides special provisions for the administration of the
tribal areas in Assam, Meghalaya, Tripura and Mizoram. Hence, statement 1 is correct. The

prelims.insightsonindia.com 32
© Insights Active Learning | All rights reserved - 156493. You may not reproduce, distribute or exploit the contents in any form without
written permission by copyright owner. Copyright infringers may face civil and criminal liability
Total Marks : 200
Online Prelims TEST - 18 (SUBJECT)
( InsightsIAS Mock Test Series for UPSC Preliminary Exam 2020 ) Mark Scored : 61.33

provisions of the schedule are as follows:

It provides for autonomous districts and autonomous regions.

The governor can increase, decrease, re-organise or alter the boundary of these districts.
Hence, statement 3 is correct.

If there are different scheduled tribes in an autonomous district, the governor may divide the
area or areas inhabited by them into autonomous regions.

Each autonomous district has a district council consisting of not more than thirty members, of
whom not more than four persons shall be nominated by the governor and the rest shall be
elected on the basis of adult suffrage. The elected members of the district council hold office
for a term of five years and nominated member hold office at the pleasure of
7 13the governor.
Hence, statement 2 is incorrect. 2 5
2
38
-
The district and the regional councils can make laws on matters
86
such as land, forests, canal
m
co of property etc.
water, Jhum cultivation, local administration, inheritance
il.
a
@ gmis/are dealing with the administration of urban
54 At the Central level, which of the following ministries
local government? 6 05
1. Ministry of Urban Development 51
2
2. Ministry of Defence
m r7
3. Ministry of Home Affairsy -
4. Ministry of Finance R
o
jo y
u n
A.M
r t statements given above is/are correct?
Which of ithe
1 and 2 only
B. 2 and 3 only
C. 1, 2 and 3 only
D. 1, 2, 3 and 4

Your Answer :
Correct Answer : C

Answer Justification :

The term ‘Urban Local Government’ in India signifies the governance of an urban area by the
people through their elected representatives. The jurisdiction of an urban local government is
limited to a specific urban area which is demarcated for this purpose by the state government.

There are eight types of urban local governments in India—municipal corporation, municipality,
notified area committee, town area committee, cantonment board, township, port trust and special
purpose agency.

prelims.insightsonindia.com 33
© Insights Active Learning | All rights reserved - 156493. You may not reproduce, distribute or exploit the contents in any form without
written permission by copyright owner. Copyright infringers may face civil and criminal liability
Total Marks : 200
Online Prelims TEST - 18 (SUBJECT)
( InsightsIAS Mock Test Series for UPSC Preliminary Exam 2020 ) Mark Scored : 61.33

The system of urban government was constitutionalized through the 74th Constitutional
Amendment Act of 1992. At the Central level, the subject of ‘urban local government’ is dealt with
by the following three ministries:

Ministry of Urban Development, created as a separate ministry in 1985. Hence, statement 1


is correct.

Ministry of Defence in the case of cantonment boards. Hence, statement 2 is correct.

Ministry of Home Affairs in the case of Union Territories. Hence, statement 3 is correct.

55 With reference to Chief Minister, consider the following statements


7 13
1. The Constitution contain specific procedure for the selection and appointment2of the Chief Minister.
2. A person who is not a member of the state legislature can be appointed 8
5
as2Chief Minister for six
months. 8 63
- from the Lower House.
3. According to the Constitution, the Chief Minister should be selected
o m
Which of the statements given above is/are correct? a il.c
A. 2 only @ gm
B. 1 and 3 only
6 05
C. 2 and 3 only
5 12
D. 1, 2 and 3 r7 m
-
Your Answer : A
R oy
Correct Answer o y
:A
nj
it u
AnswerrJustification :
M
The Constitution does not contain any specific procedure for the selection and appointment of the
Chief Minister. Article 164 only says that the Chief Minister shall be appointed by the governor.
Hence, statement 1 is incorrect.

A person who is not a member of the state legislature can be appointed as Chief Minister for six
months, within which time, he should be elected to the state legislature, failing which he ceases to
be the Chief Minister. Hence, statement 2 is correct.

According to the Constitution, the Chief Minister may be a member of any of the two Houses of a
state legislature. Usually Chief Ministers have been selected from the Lower House (legislative
assembly), but, on a number of occasions, a member of the Upper House (legislative council) has
also been appointed as Chief Minister. Hence, statement 3 is incorrect.

56 With reference to Subordinate Courts, consider the following statements


1. High Court can with draw a case pending before any subordinate court, if it involves the substantial

prelims.insightsonindia.com 34
© Insights Active Learning | All rights reserved - 156493. You may not reproduce, distribute or exploit the contents in any form without
written permission by copyright owner. Copyright infringers may face civil and criminal liability
Total Marks : 200
Online Prelims TEST - 18 (SUBJECT)
( InsightsIAS Mock Test Series for UPSC Preliminary Exam 2020 ) Mark Scored : 61.33

question of law
2. Appointments and promotion of district judges in any state shall be made by Chief Justice of High
Court.

Which of the statements given above is/are correct?


A. 1 only
B. 2 only
C. Both 1 and 2
D. Neither 1 nor 2

Your Answer :
Correct Answer : A

Answer Justification :

Articles 233 to 237 in Part VI of the Constitution make the following provisions to regulate the
13
organization of subordinate courts and to ensure their independence from the executive.
7
2
The High Court can with draw a case pending before any subordinate court, 8 if it involves the25
6 3the
- 8
substantial question of law. The case can be disposed of itself or solve question of law and
return back to the same court. Hence, statement 1 is correct. m
i l .co
Appointments and promotion of district judges in anyastate shall be made by the Governor of the
gmjurisdiction in relation to such state. Hence,
state in consultation with the high court exercising
@
statement 2 is incorrect.
6 05
5 12
7
rService
57 With reference to National Legal
- m Authority (NALSA), consider the following statements,
y
Ro aid to the poor and weaker sections of the society.
1. It is a constitutional body.
2. It provides for freey legal

u njo
t statements given above is/are correct?
Which ofrithe
M
A. 1 only
B. 2 only
C. Both 1 and 2
D. Neither 1 nor 2

Your Answer : B
Correct Answer : B

Answer Justification :

National Legal Services Authority of India was formed on 9 November 1995 under the authority of
the Legal Services Authorities Act 1987. Hence, it is a statutory body. Hence, statement 1 is
incorrect.

Its purpose is to provide free legal services to eligible candidates, and to organize Lok Adalats for
speedy resolution of cases. NALSA provides for free legal aid to the poor and weaker sections of the
society. Hence, statement 2 is correct.

prelims.insightsonindia.com 35
© Insights Active Learning | All rights reserved - 156493. You may not reproduce, distribute or exploit the contents in any form without
written permission by copyright owner. Copyright infringers may face civil and criminal liability
Total Marks : 200
Online Prelims TEST - 18 (SUBJECT)
( InsightsIAS Mock Test Series for UPSC Preliminary Exam 2020 ) Mark Scored : 61.33

58 With reference to Head of the Gram Nyayalayas, consider the following statements
1. The Nyayadhikaris who will preside over these Gram Nyayalayas are strictly judicial officers.
2. He shall be appointed by the Governor with the consultation of the concerned High Court.

Which of the statements given above is/are correct?


A. 1 only
B. 2 only
C. Both 1 and 2
D. Neither 1 nor 2

Your Answer : C
Correct Answer : A

Answer Justification :

The Gram Nyayalayas Act, 2008 has been enacted to provide for the establishment of the Gram
7
Nyayalayas at the grass roots level for the purposes of providing access to justice1to3 the citizens at
2 to any citizen
5denied
2
their doorsteps and to ensure that opportunities for securing justice are not
8
due to social, economic or other disabilities. 63
-8
.c om are strictly judicial officers and
The Nyayadhikaris who will preside over these Gram Nyayalayas
il as First Class Magistrates working
will be drawing the same salary, deriving the same powers
a
under High Courts. Hence, statement 1 is correct.m g
5 @
60 in consultation with the High Court. Hence,
He shall be appointed by the State Government
2
statement 2 is incorrect. 51 7
r
-m
oy
59 Consider the following statements
R
1. There shall be a o
j y
Council of Ministers with the Chief Minister as the head to aid and advise the
n exercise of his functions
Governor inuthe
rittendered by Ministers to the Governor shall be inquired into in any court.
2. The advice
M
Which of the statements given above is/are correct?
A. 1 only
B. 2 only
C. Both 1 and 2
D. Neither 1 nor 2

Your Answer : A
Correct Answer : A

Answer Justification :

As the Constitution of India provides for a parliamentary system of government in the states on the
Union pattern, the council of ministers headed by the chief minister is the real executive authority
in the politico administrative system of a state. The council of ministers in the states is constituted
and function in the same way as the council of ministers at the Centre.

prelims.insightsonindia.com 36
© Insights Active Learning | All rights reserved - 156493. You may not reproduce, distribute or exploit the contents in any form without
written permission by copyright owner. Copyright infringers may face civil and criminal liability
Total Marks : 200
Online Prelims TEST - 18 (SUBJECT)
( InsightsIAS Mock Test Series for UPSC Preliminary Exam 2020 ) Mark Scored : 61.33

There shall be a Council of Ministers with the Chief Minister as the head to aid and advice the
Governor in the exercise of his functions, except in so far as he is required to exercise his functions
in his discretion. Hence, statement 1 is correct.

The advice tendered by Ministers to the Governor shall not be inquired into in any court. Hence,
statement 2 is incorrect.

60 Which among the following are the initiatives with respect to skill development for minorities in
India?
1. Nai Manzil
2. USTTAD
3. Nai Roshni

Select the correct answer using the code given below.


A. 1 and 2 only
B. 1 and 3 only
1 3
C. 2 only
5 27
2
38
D. 1, 2 and 3
6
Your Answer : -8
o m
il.c
Correct Answer : D

Answer Justification : gma


@
05
1 26
All the schemes given above are initiatives with respect to skill development for minorities
in India. 7 5
m r
o y-
j
Communities.oyR
Nai Manzil Scheme is an integrated Education and Livelihood Initiative for the Minority
The scheme aims to benefit the minority youths who are school-dropouts or
i t un in the community education institutions like Madrasas and skill training along with
educated
M r
certification.

Ministry of Minority Affairs launched Up-gradation of Skills and Training in Traditional


Arts/Crafts USTTAD i.e. Development as Minority communities in India are known for their
traditional skills .

Nai Roshni a Leadership Development Programme for Minority Women in 2012-13 by


providing knowledge, tools and techniques for interacting with Government systems.

61 With reference to Legislative Council, consider the following statements


1. There are nine states with legislative councils.
2. To be eligible for membership of the Legislative council, a person must have completed the age of
35 years.
3. Legislative Councils are permanent Houses.

prelims.insightsonindia.com 37
© Insights Active Learning | All rights reserved - 156493. You may not reproduce, distribute or exploit the contents in any form without
written permission by copyright owner. Copyright infringers may face civil and criminal liability
Total Marks : 200
Online Prelims TEST - 18 (SUBJECT)
( InsightsIAS Mock Test Series for UPSC Preliminary Exam 2020 ) Mark Scored : 61.33

Which of the statements given above is/are correct?


A. 3 only
B. 1 and 3 only
C. 2 and 3 only
D. 1, 2 and 3

Your Answer : A
Correct Answer : A

Answer Justification :

India has a bicameral system i.e., two Houses of Parliament. At the state level, the equivalent of the
Lok Sabha is the Vidhan Sabha or Legislative Assembly; that of the Rajya Sabha is the Vidhan
Parishad or Legislative Council.

There are only seven such Councils across the country in the states of Andhra Pradesh, Bihar,
Jammu and Kashmir, Karnataka, Maharashtra, Telangana, and Uttar Pradesh. Hence,
7 13 statement 1
is incorrect. 52
3 82
Eligibility to become a Member of Legislative Council (MLC) to be 8 6 for membership of the
eligible
-
Legislative council, a person must be citizen of India, Must have completed the age of 30 years.
Hence, statement 2 is incorrect. .c om
a il
Legislative Councils are permanent Houses, andglike m Rajya Sabha, one-third of their members retire
@
05
every two years. Hence, statement 3 is correct.
6
5 12
7
62 Which of the following state is therfirst state to prepare the Village Panchayat Development Plan
m
(VPDP) in the country.
o y-
A. Assam njo
yR
r it u
B. Karnataka
M
C. Rajasthan
D. Telangana

Your Answer : B
Correct Answer : A

Answer Justification :

Assam is the first state to prepare the Village Panchayat Development Plan (VPDP) guideline in the
country and it is considered as the model guideline for other states. Hence, Option (a) is correct.

63 Which of the following schedule in constitution of India deals about Anti-Defection Law.

A. Fifth schedule
B. Seventh schedule
C. Ninth schedule

prelims.insightsonindia.com 38
© Insights Active Learning | All rights reserved - 156493. You may not reproduce, distribute or exploit the contents in any form without
written permission by copyright owner. Copyright infringers may face civil and criminal liability
Total Marks : 200
Online Prelims TEST - 18 (SUBJECT)
( InsightsIAS Mock Test Series for UPSC Preliminary Exam 2020 ) Mark Scored : 61.33

D. Tenth schedule

Your Answer : D
Correct Answer : D

Answer Justification :

The 52nd Amendment Act of 1985 provided for the disqualification of the members of Parliament
and the state legislatures on the ground of defection from one political party to another. For this
purpose, it made changes in four Articles1 of the Constitution and added a new Schedule (the Tenth
Schedule) to the Constitution. This act is often referred to as the ‘Anti-defection law’. Later, the
91st Amendment Act of 2003 made one change in the provisions of the Tenth Schedule. It omitted
an exception provision i.e., disqualification on ground of defection not to apply in case of split.
Hence, Option (d) is correct.

64 Which of the following is defined by The Negotiable Instruments Act, 1881?


1 3
527
A. Printing of Notes
3 82
B. Repo rate and reverse repo rate
- 86
omUnion Government and State of
C. Promissory notes
D. .c
Scope of Instrument of Accession signed between Indian
Jammu and Kashmir. ail
@ gm
Your Answer : 05
6
Correct Answer : C
5 12
7
Answer Justification : - mr
y
y Ro Act, 1881 defines promissory notes, bills of exchange and
j o
The Negotiable Instruments
n specifies penalties for bouncing of cheques and other violations with respect to
cheques. Itualso
ri t
such negotiable instruments.
M
Last year Lok Sabha has passed Negotiable Instruments (Amendment) Bill, 2017 to reduce the
number of cheque dishonour cases pending in courts.

Hence, option (c) is correct.

65 With reference to Zila Parishads, consider the following statements


1. Zila Parishad or district Panchayat is the uppermost tier of the Panchayati Raj system.
2. Zila Parishad meetings shall conduct twice in a month.

Which of the statements given above is/are correct?


A. 1 only
B. 2 only
C. Both 1 and 2
D. Neither 1 nor 2

prelims.insightsonindia.com 39
© Insights Active Learning | All rights reserved - 156493. You may not reproduce, distribute or exploit the contents in any form without
written permission by copyright owner. Copyright infringers may face civil and criminal liability
Total Marks : 200
Online Prelims TEST - 18 (SUBJECT)
( InsightsIAS Mock Test Series for UPSC Preliminary Exam 2020 ) Mark Scored : 61.33

Your Answer : A
Correct Answer : A

Answer Justification :

Zila Parishad or district Panchayat is the uppermost tier of the Panchayati Raj system. Hence,
statement 1 is correct. This institution has some directly elected members whose number differs
from State to State as it is also based on population. Chairpersons of Panchayat Samitis are ex-
officio members of Zila Parishads. Members of Parliament, Legislative Assemblies and Councils
belonging to the districts are also nominated members of Zila Parishads. The chairperson of a Zila
Parishad, called Adhyaksha or President is elected indirectly- by and from amongst the elected
members thereof. The vice-chairperson is also elected. Similarly, Zila Parishad meetings are
conducted once a month. Hence, statement 2 is incorrect. Special meetings can also be
convened to discuss special matters. Subject committees are also formed. Zila Parishad meetings
are conducted once a month. Special meetings can also be convened to discuss special matters.
Subject committees are also formed.
1 3
27
66 With reference to Jeevan Pramaan Scheme launched by Government of India,
8 25Consider the following
statements:
8 63
1. It is a biometric enabled digital service for pensioners. -
m
2. Only pensioners of Central Government can take benefit of
ilcothis facility.
.
a
gm
Which of the statements given above is /are correct?
@
A. 1 only
6 05
B. 2 only
5 12
C. Both 1 and 2
m r7
y-
D. Neither 1 nor 2
o
Your Answer :
j oyR
i t un : A
Correct Answer

Mr
Answer Justification :

Jeevan Pramaan is a biometric enabled digital service for pensioners. Pensioners of


Central Government, State Government or any other Government organization can take
benefit of this facility. Hence, statement 1 is correct and statement 2 is incorrect.

More than One Crore families in India can be classified as pensioner families, where the pension
disbursed by the various government bodies forms the basis for their income and sustainability.
There are about Fifty Lakh pensioners of the Central Government and a similar number of the
various State and UT Governments and various other government agencies. This includes
pensioners from the various public sector enterprises. In addition to this Army and Defence
Personnel drawing pension exceeds Twenty-five lakhs.

https://jeevanpramaan.gov.in/

67 Which of the following is/are the components of Digital India programme?

prelims.insightsonindia.com 40
© Insights Active Learning | All rights reserved - 156493. You may not reproduce, distribute or exploit the contents in any form without
written permission by copyright owner. Copyright infringers may face civil and criminal liability
Total Marks : 200
Online Prelims TEST - 18 (SUBJECT)
( InsightsIAS Mock Test Series for UPSC Preliminary Exam 2020 ) Mark Scored : 61.33

1. To provide universal access to mobile connectivity.


2. To improve electronics manufacturing in India.
3. To provide for electronic Delivery of Services.

Select the correct answer using the code given below.


A. 1 only
B. 2 and 3 only
C. 1 and 3 only
D. 1, 2 and 3

Your Answer :
Correct Answer : D

Answer Justification :

Nine pillars of Digital India


1 3
5 27
2
38
1. Broadband Highways
6
-8
2. Universal Access to Mobile Connectivity o m
il.c
gma
3. Public Internet Access Programme
5 @
0
26
7
4. e-Governance: ReformingrGovernment
51
through Technology
- m
oy
y R Delivery of Services
5. e-Kranti - Electronic

u njo
6.M
rit
Information for All

7. Electronics Manufacturing

8. IT for Jobs

9. Early Harvest Programmes

68 Consider the following statements:


1. The ratio of the length to the height (width) of the Indian National Flag shall be 4:3.
2. Indian National Anthem was first sung at 1896 session of the Indian National Congress

Which of the statements given above is/are correct?


A. 1 only

prelims.insightsonindia.com 41
© Insights Active Learning | All rights reserved - 156493. You may not reproduce, distribute or exploit the contents in any form without
written permission by copyright owner. Copyright infringers may face civil and criminal liability
Total Marks : 200
Online Prelims TEST - 18 (SUBJECT)
( InsightsIAS Mock Test Series for UPSC Preliminary Exam 2020 ) Mark Scored : 61.33

B. 2 only
C. Both 1 and 2
D. Neither 1 nor 2

Your Answer : C
Correct Answer : D

Answer Justification :

The National Flag shall be rectangular in shape. The ratio of the length to the height
(width) of the Flag shall be 3:2. The design of the National Flag was adopted by the Constituent
Assembly of India on July 22, 1947. Hence, statement 1 is incorrect.

The song Jana-gana-mana, composed originally in Bangla by Rabindranath Tagore, was adopted in
its Hindi version by the Constituent Assembly as the National Anthem of India on January 24, 1950.
It was first sung on December 27, 1911 at the Calcutta Session of the Indian National Congress.

7 13
The song Vande Mataram, composed in Sanskrit by Bankimchandra Chatterji,2was a source of
inspiration to the people in their struggle for freedom. It has an equal 8 25 with Jana-gana-
status
mana. The first political occasion when it was sung was the 1896 8 63 session of the Indian
-
National Congress. Hence, statement 2 is incorrect. m
i l .co
69 Consider the following statements: gma
0 5@
1. A minister belonging to the Lok Sabha can participate in the proceedings of the Rajya Sabha and
6
vice-versa.
5 12 after a new Lok Sabha has been elected is referred as
Lame-Duck session. m r7
2. The last session of the existing Lok Sabha,

o y-
j oy R given above is/are correct?
Which of the statements
A. 1 only n
B. 2 r it u
only
C.MBoth 1 and 2
D. Neither 1 nor 2

Your Answer : C
Correct Answer : C

Answer Justification :

Both statements are correct.

In addition to the members of a House, every minister and the attorney


general of India have the right to speak and take part in the proceedings of
either House, any joint sitting of both the Houses and any committee of
Parliament of which he is a member, without being entitled to vote.

Lame –Duck Session refers to the last session of the existing Lok Sabha, after a new Lok
Sabha has been elected. Those members of the existing Lok Sabha who could not get re-

prelims.insightsonindia.com 42
© Insights Active Learning | All rights reserved - 156493. You may not reproduce, distribute or exploit the contents in any form without
written permission by copyright owner. Copyright infringers may face civil and criminal liability
Total Marks : 200
Online Prelims TEST - 18 (SUBJECT)
( InsightsIAS Mock Test Series for UPSC Preliminary Exam 2020 ) Mark Scored : 61.33

elected to the new Lok Sabha are called lame-ducks.

70 With reference to the various kinds of question asked during the Question Hour, consider the
following statements:
1. A starred question requires a written answer and hence supplementary questions cannot follow.
2. An unstarred question, requires an oral answer and hence, supplementary questions can follow.
3. A short notice question is one that is asked by giving a notice of less than five days.

Which of the statements given above is/are correct?


A. 1 and 2 only
B. 3 only
C. 1, 2 and 3
D. None

Your Answer : B
Correct Answer : D 1 3
5 27
2
Answer Justification :
6 38
None of the statements given above are correct. -8
.c om
a
The first hour of every parliamentary sitting is slottedil for Question Hour. During this time,
gm give answers. The questions are of three
the members ask questions and the ministers usually
@
05notice.
kinds, namely, starred, unstarred and short
6
5 12an asterisk) requires an oral answer
r7
A starred question (distinguished by
and hence supplementary m questions can follow.
o y-
j oy R on the other hand, requires a written answer and
An unstarred question,

u n
hence, supplementary questions cannot follow.
i t
Mrnotice question is one that is asked by giving a notice of less than
A short
ten days. It is answered orally.

71 Consider the following statements:


1. Chagos islands are located to the north of Maldives.
2. U.S military base in the Diego Garcia island is located in the Chagos island archipelago.

Which of the statements given above is/are correct?


A. 1 only
B. 2 only
C. Both 1 and 2
D. Neither 1 nor 2

Your Answer :
Correct Answer : B

prelims.insightsonindia.com 43
© Insights Active Learning | All rights reserved - 156493. You may not reproduce, distribute or exploit the contents in any form without
written permission by copyright owner. Copyright infringers may face civil and criminal liability
Total Marks : 200
Online Prelims TEST - 18 (SUBJECT)
( InsightsIAS Mock Test Series for UPSC Preliminary Exam 2020 ) Mark Scored : 61.33

Answer Justification :

Chagos islands are located to the south of Maldives. Hence, statement 1 is incorrect.

3
71
U.S military base in the Diego Garcia island is located in the Chagos island archipelago.
2
Hence,
statement 2 is correct. 5 2
8
63
https://www.bbc.com/news/uk-50511847
-8
o m
72 Consider the following statements: a il.c
g m
1. The golden langur conservation project was funded by the Central Zoo authority.
5@ Protection Act, 1972
2. The golden langur is listed in Schedule I of0Wildlife
6 in IUCN Red list.
12
3. The golden langur is listed as near threatened
5
7
mr
- above is/are correct?
Which of the statements given
A. 1 and 2 only Ro
y
B. 2 only
j oy
C. 3 onlyun
it
D. 1,r2 and 3
M
Your Answer :
Correct Answer : A

Answer Justification :

Assam government has recently announced the success of the Golden Langur Conservation
Breeding in the state. This project was undertaken at the Assam State Zoo in Guwahati during the
2011-12 fiscal. It was funded by the Central Zoo authority. Hence, statement 1 is correct.

Golden Langurs are listed in Appendix I of CITES and Schedule I of Wildlife Protection Act,
1972. It is currently endangered with the current population trend as decreasing. Hence,
statement 2 is correct and statement 3 is incorrect.

73 Sadikpur Sinauli, archaeological site was recently seen in news is located in:
#118921

prelims.insightsonindia.com 44
© Insights Active Learning | All rights reserved - 156493. You may not reproduce, distribute or exploit the contents in any form without
written permission by copyright owner. Copyright infringers may face civil and criminal liability
Total Marks : 200
Online Prelims TEST - 18 (SUBJECT)
( InsightsIAS Mock Test Series for UPSC Preliminary Exam 2020 ) Mark Scored : 61.33

A. Haryana
B. Madhya Pradesh
C. Bihar
D. Uttar Pradesh

Your Answer :
Correct Answer : D

Answer Justification :

An ancient site with chariots, swords and other objects pointing to the presence of a warrior class
around 4,000 years ago in Uttar Pradesh's Baghpat district could be declared a site of national
importance soon.

Hence, option (d) is correct.

3
https://www.thehindu.com/news/national/up-site-likely-to-get-national-importance-tag/article280778
1
74.ece 2 7
2 5
6 38
74 Consider the following pairs: -8
Tribe : State o m
1. Idu Mishmi Arunachal Pradesha il.c
2. Abujh Marias Chhattisgarh
@ gm
3. Soligas 0
Karnataka 5
2 6
51
m r7
- is/are correctly matched?
Which of the pairs givenyabove
A. 1 and 2 only R o
B. 2 and 3 onlyj oy
i
C. 1 and t u3nonly
D.M
r
1, 2 and 3

Your Answer :
Correct Answer : D

Answer Justification :

All the pairs given above are correctly matched.

Idu-Mishmi tribe is the lone inhabitant tribe of Dibang Valley district. They are of mongoloid
race and have distinctive dialect of Tibeto-Burma language. Idu-Mishmi tribe can be distinctively
identified among other tribal groups of Arunachal Pradesh.

The Chhattisgarh government is processing habitat rights for Abujh Marias, a Particularly
Vulnerable Tribal Group (PVTG).

Soliga is an ethnic group of India. Its members inhabit the Biligiriranga Hills and associated

prelims.insightsonindia.com 45
© Insights Active Learning | All rights reserved - 156493. You may not reproduce, distribute or exploit the contents in any form without
written permission by copyright owner. Copyright infringers may face civil and criminal liability
Total Marks : 200
Online Prelims TEST - 18 (SUBJECT)
( InsightsIAS Mock Test Series for UPSC Preliminary Exam 2020 ) Mark Scored : 61.33

ranges in southern Karnataka, mostly in the Chamarajanagar and Erode districts of Tamil
Nadu.

75 With reference to the Sheath blight disease, consider the following statements:
1. It is caused by fungus.
2. It adversely affects the yield of paddy crop.

Which of the following statements given above is/are not correct?


A. 1 only
B. 2 only
C. Both 1 and 2
D. Neither 1 nor 2

Your Answer :
Correct Answer : D
1 3
Answer Justification : 5 27
2
6 38
-8
Both statements given are correct.
m
Sheath blight is a fungal disease caused by Rhizoctonia
i l .cosolani. Infected leaves senesce or
a destroyed. As a result, the leaf area of the
dry out and die more rapidly, young tillers can also be
gm
canopy can significantly be reduced by the disease.
0 5@
26 in rice yield.
It can cause up to 60 per cent reduction
1
r 75
y - m has released End of Childhood Index ?
76 Which of the following organizations

y Ro
A. Save the o
njChildren
u
it Rights and You
B. Child
r
C.M UNICEF
D. Pratham

Your Answer : C
Correct Answer : A

Answer Justification :

The End of Childhood Index is part of the Global Childhood Report released by UK-based Save
the Children, a non-profit organisation that works for child rights. The index evaluates
countries on eight indicators to determine the wellbeing of children and teenagers (0-19 years).

Hence, option (a) is correct.

77 With reference to the Organization of the Petroleum Exporting Countries (OPEC), consider the
following statements:

prelims.insightsonindia.com 46
© Insights Active Learning | All rights reserved - 156493. You may not reproduce, distribute or exploit the contents in any form without
written permission by copyright owner. Copyright infringers may face civil and criminal liability
Total Marks : 200
Online Prelims TEST - 18 (SUBJECT)
( InsightsIAS Mock Test Series for UPSC Preliminary Exam 2020 ) Mark Scored : 61.33

1. It is created at the Baghdad Conference in 1960’s.


2. Iraq and Iran were founding members of OPEC.
3. Recently, Qatar terminated its membership.

Which of the statements given above is/are correct?


A. 2 only
B. 1 and 3 only
C. 3 only
D. 1, 2 and 3

Your Answer : D
Correct Answer : D

Answer Justification :

All the statements given above are correct.


7 13
The Organization of the Petroleum Exporting Countries (OPEC) was founded
2 52in Baghdad, Iraq,
with the signing of an agreement in September 1960 by five countries
3 8 namely Islamic
Republic of Iran, Iraq, Kuwait, Saudi Arabia and Venezuela. 8 6
- They were to become the
om
Founder Members of the Organization.

il .c
m a
These countries were later joined by Qatar (1961), Indonesia (1962), Libya (1962), the United Arab
g
Emirates (1967), Algeria (1969), Nigeria (1971), Ecuador (1973), Gabon (1975), Angola (2007),
Equatorial Guinea (2017) and Congo (2018). 0 5@
1 26
Ecuador suspended its membership r 75in December 1992, but rejoined OPEC in October 2007.
y -m
Indonesia suspended its membership in January 2009, reactivated it again in January 2016, but
decided to suspend its o st
membership once more at the 171 Meeting of the OPEC Conference on 30
y R
November 2016.oGabon terminated its membership in January 1995. However, it rejoined the
Organizationu ninjJuly 2016. Qatar terminated its membership on 1 January 2019.
r it
M
This means that, currently, the Organization has a total of 14 Member Countries.

https://www.opec.org/opec_web/en/about_us/25.htm

https://www.thehindubusinessline.com/markets/commodities/oil-market-will-be-oversupplied-despite
-opec-pact-says-iea/article30287746.ece

78 World Economic Outlook, sometime seen in the news, is published by

A. World Bank
B. World Economic Forum
C. International Monetary Fund
D. None of the above

Your Answer : C
Correct Answer : C

prelims.insightsonindia.com 47
© Insights Active Learning | All rights reserved - 156493. You may not reproduce, distribute or exploit the contents in any form without
written permission by copyright owner. Copyright infringers may face civil and criminal liability
Total Marks : 200
Online Prelims TEST - 18 (SUBJECT)
( InsightsIAS Mock Test Series for UPSC Preliminary Exam 2020 ) Mark Scored : 61.33

Answer Justification :

The World Economic Outlook (WEO) is a report by the International Monetary Fund that
analyzes key parts of the IMF's surveillance of economic developments and policies in its
member countries. It also projects developments in the global financial markets and economic
systems.

79 With reference to Anthrax, consider the following statements


1. Anthrax is a disease caused by Bacillus anthracis
2. It can be spread directly from one infected animal or person to another.

Which of the statements given above is/are correct?


A. 1 only
B. 2 only
C. Both 1 and 2
D. Neither 1 nor 2
1 3
5 27
2
Your Answer :
6 38
-8
Correct Answer : A

om
il.c
Answer Justification :

m a
Anthrax is a serious infectious disease caused byggram-positive, rod-shaped bacteria known as
@
6 05
Bacillus anthracis. Anthrax can be found naturally in soil and commonly affects domestic and wild
2
animals around the world. Hence, statement
11 is correct.
5
r7
m from one infected animal or person to another; it is spread
Anthrax does not spread directly
-
y contracted in laboratory accidents or by handling infected animals or
by spores. Anthrax canobe
R
y Hence, statement 2 is incorrect.
their wool or hides.

u njo
r it
M
80 With reference to AWaRe, consider the following statements
1. This global campaign has been started by Center for a Livable Future (CLF).
2. It aims at guiding policy-makers and health workers to use antibiotics safely and more effectively.

Which of the statements given above is/are correct?


A. 1 only
B. 2 only
C. Both 1 and 2
D. Neither 1 nor 2

Your Answer :
Correct Answer : B

Answer Justification :

AWaRe is an online tool aimed at guiding policy-makers and health workers to use antibiotics
safely and more effectively. Hence, statement 2 is correct.

prelims.insightsonindia.com 48
© Insights Active Learning | All rights reserved - 156493. You may not reproduce, distribute or exploit the contents in any form without
written permission by copyright owner. Copyright infringers may face civil and criminal liability
Total Marks : 200
Online Prelims TEST - 18 (SUBJECT)
( InsightsIAS Mock Test Series for UPSC Preliminary Exam 2020 ) Mark Scored : 61.33

The tool, known as ‘AWaRe’, classifies antibiotics into three groups:

Access — antibiotics used to treat the most common and serious infections.

Watch — antibiotics available at all times in the healthcare system.

Reserve — antibiotics to be used sparingly or preserved and used only as a last resort.

World Health Organization (WHO) has launched this global campaign. Hence, statement 1 is
incorrect.

81 With reference to One Nation One Ration Card, consider the following statements
1. Launched by Ministry of Consumer Affairs, Food and Public Distribution
7 13
52nation from any PDS
2. It will allow portability of food security benefits
8
3. It will ensure all beneficiaries especially migrants can access PDS across 2
the
shop of their own choice. 63
-8
m
Which of the statements given above is/are correct?
i l .co
ma
A. 2 only
B. 1 and 3 only g
C. 2 and 3 only 0 5@
6
D. 1, 2 and 3 12 5
7
Your Answer : D - mr
y
Correct Answer : D
y Ro
j o
i t un
Answer Justification :

Mr
One Nation One Ration Card (RC) will ensure all beneficiaries especially migrants can access PDS
across the nation from any PDS shop of their own choice. Hence, statement 3 is correct.

It will allow portability of food security benefits. To lift their entitlement food grains from any Fair
Price Shop in the country. Hence, statement 2 is correct.

‘One Nation One Ration Card’ scheme is launched by Ministry of Consumer Affairs, Food and
Public Distribution. It will be available across the country from July 1, 2020. Hence, statement 1
is correct.

82 The Bangkok Declaration on Combating Marine Debris has been adopted by

A. G20
B. SAARC
C. BRICS
D. ASEAN

prelims.insightsonindia.com 49
© Insights Active Learning | All rights reserved - 156493. You may not reproduce, distribute or exploit the contents in any form without
written permission by copyright owner. Copyright infringers may face civil and criminal liability
Total Marks : 200
Online Prelims TEST - 18 (SUBJECT)
( InsightsIAS Mock Test Series for UPSC Preliminary Exam 2020 ) Mark Scored : 61.33

Your Answer :
Correct Answer : D

Answer Justification :

The Bangkok Declaration on Combating Marine Debris in ASEAN Region was adopted by leaders of
the 10-member Association of Southeast Asian Nations (ASEAN), which includes four of the world’s
top polluters. Which promises to “prevent and significantly reduce marine debris. Neither the
declaration nor its accompanying Framework of Action specifically mention bans on single-use
plastic or imports of foreign waste, as environmental groups previously demanded ahead of the
summit. Hence, option (d) is correct.

83 With reference to Indian grey wolf, consider the following statements


1. It inhabits semi-arid and arid areas.
2. It has a wide distribution range that extends from the Indian subcontinent to Israel.
3. The IUCN listed this as Vulnerable in its Red list. 3 1
7
252
Which of the statements given above is/are correct? 8
63
-8
A. 2 only
B. 1 and 3 only
o m
il.c
C. 1 and 2 only
D. 1, 2 and 3
gma
5 @
Your Answer : 0
Correct Answer : C 126
5
m r7
Answer Justification : -
oy
j oy Rwolf to be seen in Bangladesh in eight decades has been beaten to death by
The first Indian grey
farmers aftern preying on their livestock, wildlife experts.
rit u
M
It is a subspecies of grey wolf. It inhabits semi-arid and arid areas. Hence, statement 1 is correct.

It has a wide distribution range that extends from the Indian subcontinent to Israel. Hence,
statement 2 is correct.

The IUCN Redlist considering the wide distribution range of the subspecies has listed it as a
species of least concern. Hence, statement 3 is incorrect.

84 consider the following statements


1. RailWire is a retail Broadband initiative of the RailTel
2. RailTel Corporation is a Navartna PSU of Ministry of Railways.

Which of the statements given above is/are correct?


A. 1 only
B. 2 only
C. Both 1 and 2

prelims.insightsonindia.com 50
© Insights Active Learning | All rights reserved - 156493. You may not reproduce, distribute or exploit the contents in any form without
written permission by copyright owner. Copyright infringers may face civil and criminal liability
Total Marks : 200
Online Prelims TEST - 18 (SUBJECT)
( InsightsIAS Mock Test Series for UPSC Preliminary Exam 2020 ) Mark Scored : 61.33

D. Neither 1 nor 2

Your Answer :
Correct Answer : A

Answer Justification :

RailWire is a retail Broadband initiative of the RailTel. It envisages extending broadband and
application services to the public. The Wi-Fi at stations has been provided in association with
Google as the technology partner. Hence, statement 1 is correct.

RailTel Corporation is a “Mini Ratna(Category-I)” PSU of Ministry of Railways. It is the largest


neutral telecom services providers in the country. Hence, statement 2 is incorrect.

85 With reference to Comprehensive Convention on International Terrorism (CCIT), consider the


following statements
1. It intends to criminalize all forms of international terrorism 7 13
2. It was proposed by India 2 52
8
3. Recently, it was adopted by UN 63
-8
Which of the statements given above is/are correct?
.c om
A. 2 only a il
B. 1 and 2 only
@ gm
C. 2 and 3 only
6 05
D. 1, 2 and 3 2 1
5
Your Answer : m r7
-
Correct Answer : B oy
j oyR
u n
Answer Justification :
i t
Mr
The Comprehensive Convention on International Terrorism is a proposed treaty which intends to
criminalize all forms of international terrorism and deny terrorists, their financiers and supporters
access to funds, arms, and safe havens. Hence, statement 1 is correct.

It is a draft proposed by India in 1996. Hence, statement 2 is correct.

This proposed treaty NOT adopted yet by UN. Hence, statement 3 is incorrect.

86 Which of the following country became the world's first to successfully perform a telerobotic
coronary intervention?

A. Japan
B. China
C. India
D. USA

prelims.insightsonindia.com 51
© Insights Active Learning | All rights reserved - 156493. You may not reproduce, distribute or exploit the contents in any form without
written permission by copyright owner. Copyright infringers may face civil and criminal liability
Total Marks : 200
Online Prelims TEST - 18 (SUBJECT)
( InsightsIAS Mock Test Series for UPSC Preliminary Exam 2020 ) Mark Scored : 61.33

Your Answer :
Correct Answer : C

Answer Justification :

India became the world's first to successfully perform a telerobotic coronary intervention on
December 5 by an Ahmedabad base cardiologist in Gujarat. It is robotic method of performing heart
surgery. With the help of the internet and a robotic tower, a surgeon is able to treat patients from a
distance. Hence, option (C) is correct.

87 Kapu is a social grouping of agriculturists found primarily in

A. Tamil Nadu
B. Kerala
C. Karnataka
D. Andhra Pradesh
1 3
5 27
2
38
Your Answer :
6
-8
Correct Answer : D

om
il.c
Answer Justification :

m a
Kapu literally means cultivator or agriculturist ingTelugu. It is a dominant peasant caste found in
@
5 and Telangana. Hence, option (d) is correct.
Telugu speaking states of Andhra Pradesh0(AP)
2 6
7 51
88 With reference to Generalized m r of Preferences (GSP), consider the following statements.
System
1. It is a preferential tariff y
-
system extended by developed countries
o
y R has extended GSP status to India
2. Currently 60 countries

u njo
t statements given above is/are correct?
Which ofrithe
M
A. 1 only
B. 2 only
C. Both 1 and 2
D. Neither 1 nor 2

Your Answer : C
Correct Answer : A

Answer Justification :

The Generalized System of Preferences (GSP) provides duty-free treatment to goods of designated
beneficiary countries. The program was authorized by the Trade Act of 1974 to promote economic
growth in the developing countries and was implemented on January 1, 1976. It is a preferential
tariff system extended by developed countries. Hence, statement 1 is correct.

Presently 28 countries has extended GSP status to India.These are Japan, NZ, Canada, Russia, 17
EU countries etc. Hence, statement 2 is incorrect.

prelims.insightsonindia.com 52
© Insights Active Learning | All rights reserved - 156493. You may not reproduce, distribute or exploit the contents in any form without
written permission by copyright owner. Copyright infringers may face civil and criminal liability
Total Marks : 200
Online Prelims TEST - 18 (SUBJECT)
( InsightsIAS Mock Test Series for UPSC Preliminary Exam 2020 ) Mark Scored : 61.33

89 With reference to Swachh Survekshan 2020 league, consider the following statements.
1. It is part of annual Swachh Survekshan
2. This was launched by Ministry of Drinking Water and Sanitation

Which of the statements given above is/are correct?


A. 1 only
B. 2 only
C. Both 1 and 2
D. Neither 1 nor 2

Your Answer :
Correct Answer : A

Answer Justification :

The Swachh Survekshan League will be a quarterly cleanliness assessment of Indian cities and
7 13 Survekshan
towns. It will be integrated with Swachh Survekshan 2020, the 5th edition of Swachh
2 522020. Hence,
survey of urban India, which will be conducted between January and February
statement 1 is correct. 38 6
-8
.c om Urban Affairs Ministry. Hence,
Swachh Survekshan 2020 League was launched by Housing and
statement 2 is incorrect. il
gma
@
90 With reference to Bharat bill payment system
6 05(BBPS), consider the following statements.
12in India offering interoperable and accessible bill payment
1. It is an integrated bill payment system
5
service to customers
m r7
2. It is owned by Google -
R oy
j oy
Which of the statements given above is/are correct?
n
A. 1 only
r i tu
B.M2 only
C. Both 1 and 2
D. Neither 1 nor 2

Your Answer :
Correct Answer : A

Answer Justification :

BBPS is an integrated bill payment system in India offering interoperable and accessible bill
payment service to customers. It is a one-stop payment platformfor all bills, providing an
interoperable and accessible “Anytime Anywhere” bill payment service to customers across the
country with certainty, reliability and safety of transactions. Hence, statement 1 is correct.

The Bharat Bill Payment System was launched by the National Payment Corporation of India on
31 August 2016 under the recommendation of RBI executive director G. Padmanabhan committee.
Hence, statement 2 is incorrect.

prelims.insightsonindia.com 53
© Insights Active Learning | All rights reserved - 156493. You may not reproduce, distribute or exploit the contents in any form without
written permission by copyright owner. Copyright infringers may face civil and criminal liability
Total Marks : 200
Online Prelims TEST - 18 (SUBJECT)
( InsightsIAS Mock Test Series for UPSC Preliminary Exam 2020 ) Mark Scored : 61.33

91 With reference to Union for the Protection of New Varieties of Plants (UPOV), consider the following
statements.
1. It aims to provide and promote an effective system of plant variety protection
2. India is yet to join this organization.

Which of the statements given above is/are correct?


A. 1 only
B. 2 only
C. Both 1 and 2
D. Neither 1 nor 2

Your Answer :
Correct Answer : A

Answer Justification :

1
The International Union for the Protection of New Varieties of Plants (UPOV) is7an
3
2 the Protection of
intergovernmental organization established by the International Convention5for
New Varieties of Plants. The convention was adopted in Paris in 1961 3 82it was revised in 1972,
and
6
- 8of plant variety protection, with
1978 and 1991. It aims to provide and promote an effective system
m for the benefit of society.
the aim of encouraging the development of new varieties ofoplants,
Hence, statement 1 is correct. il .c
gma
5@statement 2 is incorrect.
In 2002, India joined this organization. Hence,
0
1 26
r 75 stone for a memorial to mark completion of 200 years of
92 President has recently laid the foundation
Paika Rebellion, in this contextm
y - consider the following statements.
Ro
1. The Paiks were the traditional landed militia in the state of Jharkhand
2. The Paikas rose inyarmed rebellion in 1817 under the leadership of Baxi Jagabandhu.
u njo
t
Which ofrithe statements given above is/are correct?
M
A. 1 only
B. 2 only
C. Both 1 and 2
D. Neither 1 nor 2

Your Answer : B
Correct Answer : B

Answer Justification :

The Paiks were the traditional landed militia of Khurda, Odisha. Rulers of Khurda were traditionally
the custodians of Jagannath Temple and ruled as the deputy of Lord Jagannath on earth. The
British, having established their sway over Bengal Province occupied the Khurda’s land in 1803 and
took over the hereditary rent-free lands granted to Paikas. Hence, statement 1 is incorrect.

The Paikas rose in armed rebellion in 1817 under the leadership of Baxi Jagabandhu. The tribals of
Ghumusar and other sections of the population actively took part in it. Hence, statement 2 is

prelims.insightsonindia.com 54
© Insights Active Learning | All rights reserved - 156493. You may not reproduce, distribute or exploit the contents in any form without
written permission by copyright owner. Copyright infringers may face civil and criminal liability
Total Marks : 200
Online Prelims TEST - 18 (SUBJECT)
( InsightsIAS Mock Test Series for UPSC Preliminary Exam 2020 ) Mark Scored : 61.33

correct.

93 With reference to SEAD initiative, consider the following statements


1. It is a voluntary collaboration among governments to promote use of energy efficient appliances
worldwide by Clean Energy Ministerial.
2. Currently, there are 118 participating governments to this initiative.

Which of the statements given above is/are correct?


A. 1 only
B. 2 only
C. Both 1 and 2
D. Neither 1 nor 2

Your Answer :
Correct Answer : A
1 3
Answer Justification : 527
2
The Super-efficient Equipment and Appliance Deployment (SEAD) 8
38
of6CEM is a voluntary
-
collaboration among governments to promote use of energy efficient appliances worldwide. It is a
m
oCooperation
task of the International Partnership for Energy Efficiency
i l .c (IPEEC). Hence, statement
1 is correct. a
@ gm
5
60 energy efficiency standards, 2,000 TWh of annual
Presently, there are 18 participating governments to this initiative. If all SEAD governments were to
adopt current policy best practices for2product
1
5equal
electricity could be saved in 2030,
r 7 to the energy generated by 650 mid-sized power plants.
-m
Hence, statement 2 is incorrect.
y
y Ro
94 The Virtual Policejo
u n Station initiative has been started by
r it
M
A. Karnataka
B. Uttar Pradesh
C. Kerala
D. Andhra Pradesh

Your Answer : D
Correct Answer : D

Answer Justification :

The Virtual Police Station is an initiative by the Andhra Pradesh government. It is to demystify the
police station and acquaint the public with the functioning of a police station electronically without
visiting it physically. It will set up in the educational institutions as a pilot project and it will educate
public about the functioning of police stations and how to file a FIR. Hence, option (D) is correct.

95 The Exercise INDRA joint Tri-services exercise between

prelims.insightsonindia.com 55
© Insights Active Learning | All rights reserved - 156493. You may not reproduce, distribute or exploit the contents in any form without
written permission by copyright owner. Copyright infringers may face civil and criminal liability
Total Marks : 200
Online Prelims TEST - 18 (SUBJECT)
( InsightsIAS Mock Test Series for UPSC Preliminary Exam 2020 ) Mark Scored : 61.33

A. India-china
B. India- USA
C. India- Russia
D. India- Nepal

Your Answer :
Correct Answer : C

Answer Justification :

Exercise INDRA is a joint Tri-services exercise between India and Russia. The Indra series of
exercise began in 2003 and the first joint Tri Services Exercise was conducted in 2017. INDRA 2019
will be conducted simultaneously at Babina (near Jhansi), Pune, and Goa. Hence, option (C) is
correct.

96 With reference to Payments banks, consider the following statements


7 13
1. They can undertake lending activities.
2 52
2. They should maintain Cash Reserve Ratio (CRR) with RBI.
6 38
3. They can issue both Debit cards and credit cards 8 -
om
il.c
Which of the statements given above is/are correct?
A. 2 only a
B. 1 and 3 only
@ gm
C. 2 and 3 only
6 05
D. 1, 2 and 3 12 5
7
Your Answer : A - mr
y
Correct Answer : A
y Ro
n j o
u
Answer Justification
it
:
M r
Payments banks is a new model of banks conceptualized by the Reserve Bank of India (RBI). These
banks can accept a restricted deposit, which is currently limited to ₹100,000 per customer and may
be increased further. They cannot undertake lending activities. Hence, statement 1 is incorrect.

They should maintain Cash Reserve Ratio (CRR) with RBI. It is required to invest minimum 75% of
its "demand deposit balances" in Statutory Liquidity Ratio (SLR). Hence, statement 2 is correct.

They can issue ATM/Debit cards however cannot issue credit cards. Hence, statement 3 is
incorrect.

97 Consider the following statements


1. Skills Build platform is a digital platform developed by IBM along with Ministry of Skill
Development.
2. The platform enables holistic learning and aligning it with Skills India initiative by Government of
India.

prelims.insightsonindia.com 56
© Insights Active Learning | All rights reserved - 156493. You may not reproduce, distribute or exploit the contents in any form without
written permission by copyright owner. Copyright infringers may face civil and criminal liability
Total Marks : 200
Online Prelims TEST - 18 (SUBJECT)
( InsightsIAS Mock Test Series for UPSC Preliminary Exam 2020 ) Mark Scored : 61.33

Which of the statements given above is/are correct?


A. 1 only
B. 2 only
C. Both 1 and 2
D. Neither 1 nor 2

Your Answer :
Correct Answer : C

Answer Justification :

Skills Build is a digital platform developed by IBM along with Ministry of Skill Development. India is
the 4th country where this Skills Build platform has been launched after in UK, Germany and
France. Hence, statement 1 is correct.

The platform enables holistic learning and aligning it with Skills India initiative by Government of
13 for
India. It will be accessible online through bharat skills, a Learning Management System
7
ITI/NSTI Students and Teachers and free to use. It will focus on personalised 2
2 5 coaching and
experiential learning. It also provides personal assessment for cognitive8capabilities. Hence,
3
statement 2 is correct. 86 -
.c om
a
98 With reference to International Geological Congress (IGC),il consider the following statements.
1. India is the only Asian country to host the event gm
twice.
@
05 for organizing the event.
2. Geological Survey of India is the nodal agency
6
2
51
7 is/are correct?
Which of the statements given above
r
A. 1 only
y -m
Ro
B. 2 only
C. Both 1 and 2y
jo
un1 nor 2
D. Neither
it
M r
Your Answer :
Correct Answer : C

Answer Justification :

India is gearing up to host the 36th International Geological Congress (IGC) in March, 2020. India is
the only Asian country to host the event twice. In 1964, India had hosted it for the first time, which
was the 22nd IGC. Geological Survey of India is the nodal agency for organizing the event. Hence,
both statement 1 and statement 2 are correct.

It is a global geoscientific events held once in four years and participated by around 5000-6000
geoscientists from all across the world.

99 With reference to Trakea, consider the following statements


1. It is an unique barcoding software adopted by Assam Police.
2. Trakea ensures foolproof security of the samples collected from the scene of crime, and the forensic

prelims.insightsonindia.com 57
© Insights Active Learning | All rights reserved - 156493. You may not reproduce, distribute or exploit the contents in any form without
written permission by copyright owner. Copyright infringers may face civil and criminal liability
Total Marks : 200
Online Prelims TEST - 18 (SUBJECT)
( InsightsIAS Mock Test Series for UPSC Preliminary Exam 2020 ) Mark Scored : 61.33

analysis reports.

Which of the statements given above is/are correct?


A. 1 only
B. 2 only
C. Both 1 and 2
D. Neither 1 nor 2

Your Answer :
Correct Answer : B

Answer Justification :

It is a unique barcoding software adopted by Haryana Police. Trakea ensures foolproof security of
the samples collected from the scene of crime, and the forensic analysis reports. Hence, statement
1 is incorrect and statement 2 is correct.
7 13
Haryana Police claims it is the country’s first police force to have introduced
2 52this unique barcoding
for forensic reports. It aims to ensure that thousands of forensic reports8that form the backbone of
the criminal investigation system and subsequent trials in the courts 8 6of3law, are not tampered with.
-
m
i l .coconsider the following statements.
mafrom financial sector, capital market,
100 With reference to Serious Fraud Investigation Office (SFIO),
g
1. It is a multi-disciplinary organization having experts
5@
accountancy, forensic audit, taxation, law,0information technology, company law, customs and
investigation. 2 6
2. It is under the jurisdiction of the 7 51 of Finance, Government of India.
Ministry
- mr
Which of the statements
R oygiven above is/are correct?
A. 1 only
j oy
B. 2 onlyun
r
C. Bothit1 and 2
D.M Neither 1 nor 2

Your Answer :
Correct Answer : A

Answer Justification :

It is a fraud investigating agency. The SFIO is involved in major fraud probes and is the co-
ordinating agency with the Income Tax and CBI. It is a multi-disciplinary organization having
experts from financial sector, capital market, accountancy, forensic audit, taxation, law, information
technology, company law, customs and investigation. These experts have been taken from various
organizations like banks, Securities and Exchange Board of India, Comptroller and Auditor General
and concerned organizations and departments of the Government. Hence, statement 1 is correct.

It is under the jurisdiction of the Ministry of Corporate Affairs, Government of India. Hence,
statement 2 is incorrect.

prelims.insightsonindia.com 58
© Insights Active Learning | All rights reserved - 156493. You may not reproduce, distribute or exploit the contents in any form without
written permission by copyright owner. Copyright infringers may face civil and criminal liability

You might also like